Midterm

Pataasin ang iyong marka sa homework at exams ngayon gamit ang Quizwiz!

Parkinson Drugs Please select the drug name/category that matches each mechanism of action: 1. Prolongs dopamine effects by inhibiting MAO; possible neuroprotective effects 2. Antiviral agent; blocks NMDA receptors in the brain to reduce excitatory response 3. Decreases acetylcholine influence 4. Inhibits dopa decarboxylase; prevents premature conversion a. Carbidopa b. Benztropine c. Selegiline d. Amantadine

1. Carbidopa - inhibits dopa decarboxylase; prevents premature conversion 2. Anticholinergic agents (benztropine) - decreases acetylcholine influence 3. Selegiline - prolongs dopamine effects by inhibiting MAO; possible neuroprotective effects 4. Amantadine - antiviral agent; blocks NMDA receptors in brain

Seizures Please select the drug name/category that matches each mechanism of action: 1. Decreases Na+ entry into rapidly firing neurons 2. Decreases Ca++ entry into thalamic neurons 3. Increases GABA inhibition 4. Decreases release or effects of excitatory amino acids a. Ethosuximide b. Phenytoin c. Gabapentin d. Benzodiazepines

1. Decreases Na+ entry into rapidly firing neurons - b. Phenytoin 2. Decreases Ca++ entry into thalamic neurons - a. Ethosuximide 3. Increases GABA inhibition - d. Benzodiazepines 4. Decreases release or effects of excitatory amino acids - c. Gabapentin - this is debatable. I'll change the wording around

Muscle relaxants Please match the pharmaceutical and possible side effects 1. Diazepam 2. Gabapentin 3. Dantrolene 4. Botulinum toxin a. Dizziness and ataxia b. Local irritation at injection site and potential for antibody production c. Possible liver toxicity d. Sedation

1. Diazepam - d. Sedation 2. Gabapentin - a. Dizziness and ataxia 3. Dantrolene - c. Possible liver toxicity 4. Botulinum toxin - b. Local irritation at injection site and potential for antibody production

Anesthesia Match the anesthetic with likely indication 1. Dissociative anesthetic 2. Bier block 3. Continuous nerve block 4. Benzodiazepine a. Total knee replacement b. Post-op total knee replacement c. Complex regional pain syndrome d. Diagnostic/radiologic procedures

1. Dissociative anesthetic - d. Diagnostic/radiologic procedures 2. Bier block - c. Complex regional pain syndrome 3. Continuous nerve block - b. Post-op total knee replacement 4. Benzodiazepine - a. Total knee replacement - maybe true but confusing with the other choices. I'll change the wording.

Psychotropics For numbers 1-2, match each mechanism of action with its correct category of drug: 1. Prolong the effects of amine neurotransmitter by inhibiting reuptake or decreasing breakdown 2. Bind to GABA receptors in order to increase their effect For numbers 3-4, match the adverse side effects/overdose signs with the corresponding drug: 3. Metallic taste, fasciculations, hallucinations 4. Tardive dyskinesia, pseudoparkinsonism, akathisia (RLS) a. Lithium b. Anti-depressants c. Sedative-hypnotic; Anti-anxiety d. Anti-psychotics

1. Prolong the effects of amine neurotransmitter by inhibiting reuptake or decreasing breakdown - b. Anti-depressants 2. Bind to GABA receptors in order to increase their effect - c. Sedative-hypnotic; Anti-anxiety 3. Metallic taste, fasciculations, hallucinations - a. Lithium 4. Tardive dyskinesia, pseudoparkinsonism, akathisia (RLS) - d. Anti-psychotics

You are treating in the spinal cord unit of a rehabilitation hospital and are working on transfers with your 18 year-old patient, Mrs. Risk-Taker. Your patient has 5/5 strength of all muscles of the upper extremity. She has 5/5 strength with hip flexion, 5/5 strength with knee extension, 3/5 strength in dorsiflexion, and 0/5 strength in all muscles of the foot and ankle. She has full sensation above her knees, on her medial shins and malleoli, sacral sparing, and impaired light touch in her feet. What is her neurological level on the ASIA scale?: A. A L3 B. B L4 C. C L3 D. D L4 E. B L3

A. A L3. Injury is not complete due to sacral sparing. L3 is not the neurological level because there is ■ strength in dorsiflexion. B. B L4- CORRECT- Injury is incomplete due to sacral sparing. L4 is the neurological level because there is ■ strength in dorsiflexion and 5/5 strength in key muscles above. C. C. L3. C is incorrect because motor function is not preserved below the neurological level. D. D. L4. L4 is correct, but motor function is not preserved below the neurological level and none of key muscles below the level have a grade ≥ 3. E. B. L3. L3 is not the neurological level because there is ■ strength in dorsiflexion.

You are completing your treatment note following your evaluation of Othello, who recently sustained a SCI secondary to stab wounds. You decide to rate his impairments according to the ASIA Scale. Below are your evaluation findings: • Sensation intact bilaterally • Sensation and motor intact in S4-5 • Hip flexor strength: 5/5 on R, 5/5 on L • Knee extension strength: 4/5 on R, 5/5 on L • Dorsiflexion strength: 2/5 on R, 2/5 on L • Hallux extension strength: 2/5 on R, 2/5/ on L • Plantarflexion strength: 1/5 on R, 1/5 on L What would you rate this patient on the ASIA Scale? A. A: Complete B. B: Incomplete C. C: Incomplete D. D: Incomplete E. E: Normal

A. A: Complete - No motor or sensory function would be present below the neurological level (L3 in this case) if this classification was correct. B. B: Incomplete - No motor function would be present below the neurological level (L3 in this case) if this classification was correct. C. C: Incomplete - CORRECT - Sensory and motor function is present below the lesion; however, less than half of the major muscle groups have less than or equal to a 3/5 for strength. To be a B, no motor function would be present below the lesion. To be a D, half or more of the major muscle groups must have 3/5 strength or greater. D. D: Incomplete - Over half of the key muscle groups must have greater than 3/5 strength to qualify for this level. E. E: Normal - the patient must have normal motor and sensory function for this classification.

You are evaluating a Mr. Wile E. Coyote, who recently sustained a SCI at the level of C5 secondary to dropping an anvil on his head. He plans to file a lawsuit against the ACME corporation after he is discharged from acute care PT. During his eval, you notice that he lacks pain sensation and demonstrates increased spasticity and tone bilaterally below the lesion. Curiously (or perhaps not), when you move his arms and legs around, he is still able to tell you where you are moving his limbs in space. Furthermore, he noticed that he woke up one morning after his cell phone (set to vibrate mode) went off in his hand. With what type of SCI does Mr. Coyote present? A. Anterior Cord Syndrome B. Central Cord Syndrome C. Conus Medullaris Syndrome D. Cauda Equina Syndrome E. Dorsal Column Syndrome

A. Anterior Cord Syndrome -CORRECT-This condition impacts the anterior corticospinal and spinothalamic tracts. However, the dorsal cord remains intact and vibration and position senses are preserved. B. Central Cord Syndrome: Most commonly occurs with a hyperextension injury that compresses the cord in a narrowed canal, resulting in UMN signs below the level of the lesion (increased LE DTRs) and LMN signs at the level of the lesion (flaccid arm weakness). C. Conus Medullaris Syndrome: This syndrome affects the sacral spinal cord and lumbar nerve roots within the spinal cord and causes early sphincter dysfunction, symmetrical saddle anesthesia, variable reflexes D. Cauda Equina Syndrome: This condition is caused by an injury to the lumbosacral nerve roots, anywhere from L1 to S1. Effects include asymmetric pain that follows a dermatomal distribution, bilateral LE weakness, saddle anesthesia and late sphincter dysfunction E. Dorsal Column Syndrome: Patients with this condition will present with loss of proprioception, vibration, and position sense while pain and temperature sensation are preserved

Mr. Jones is a frequent flyer at YouDee Hospital and is now on your patient caseload. When you review his chart you see Tabes Dorsalis (neurosyphilis) in his PMH and when you go into examine him he presents with loss of proprioception, vibration and position sense while his motor function, pain, and temperature responses are all intact. However, when you get Mr. Jones up to walk to the bathroom, you see that his gait is very uncoordinated. These signs and symptoms are consistent with which spinal cord syndrome? a. Anterior Cord Syndrome b. Central Cord Syndrome c. Conus Medullaris Syndrome d. Cauda Equina Syndrome e. Dorsal Column Syndrome

A. Anterior Cord Syndrome: This condition impacts the anterior corticospinal and spinothalamic tracts. However, the dorsal cord remains intact and vibration and position senses are preserved. B. Central Cord Syndrome: Most commonly occurs with a hyperextension injury that compresses the cord in a narrowed canal, resulting in UMN signs below the level of the lesion (increased LE DTRs) and LMN signs at the level of the lesion (flaccid arm weakness). C. Conus Medullaris Syndrome: This syndrome affects the sacral spinal cord and lumbar nerve roots within the spinal cord and causes early sphincter dysfunction, symmetrical saddle anesthesia, variable reflexes D. Cauda Equina Syndrome: This condition is caused by an injury to the lumbosacral nerve roots, anywhere from L1 to S1. Effects include asymmetric pain that follows a dermatomal distribution, bilateral LE weakness, saddle anesthesia and late sphincter dysfunction E. Dorsal Column Syndrome -CORRECT- Syphillis will result in pure dorsal column dyfunction and this pt. will present with loss of proprioception, vibration, and position sense while pain and temperature sensation are preserved.

One of your patients, Homer Simpson, has begun to present with diminished proprioception and has been tripping more often. He is also losing the ability to identify small objects within either hand. While he does not recall any traumatic injuries occurring prior to the onset of these symptoms, he does remark that he has been going to Moe's Bar very often. What may be a cause of his symptoms with relation to the spinal cord? A. Anterior cord syndrome B. ALS C. Brown Sequard Syndrome D. Complete spinal cord transection E. Metabolic myopathy

A. Anterior cord syndrome - this syndrome leads to deficits in the anterior spinal cord, but would usually not impact the dorsal column, which is related to proprioception and fine touch sensation. B. ALS - ALS is related to motor function and does not lead to sensory or proprioceptive deficits. C. Brown Sequard Syndrome - This would lead to motor and proprioceptive deficits on the injured side and sensory deficits on the uninjured side. It would not generally lead to bilateral proprioceptive and fine sensation losses with normal motor activity as seen in this patient. D. Complete spinal cord transection - Homer would not have any motor or sensory activity below the neurologic level and would present much differently if this were the case. E. Metabolic myelopathy - CORRECT - He may be demonstrating a metabolic myelopathy secondary to vitamin B12 and folate deficiencies in part due to excessive alcohol consumption. This condition commonly affects the dorsal column, leading to his symptoms, as well as anemia.

You are covering for another therapist at a local outpatient physical therapy facility. You are treating John, who is 60 years old and has a note in his chart that he is unable to say anything other than 'yes,' but is able to follow complex commands. He arrives early with cupcakes his wife made. The receptionist jokingly remarks that he should grab mugs from the cabinet and pour two glasses of milk, which he does promptly. John eats his cupcake and licks the frosting off his nose when the receptionist points out he has frosting on his face. You suspect he is affected by _______. a. Broca's Aphasia b. Wernicke's Aphasia c. Parkinson's Disease d. Dysarthria e. Apraxia

A. Broca's Aphasia - CORRECT - he has difficulty expressing himself but appears to have no cognitive impairments B. Wernicke's Aphasia - Patients with Wernicke's aphasia generally do not have trouble speaking, but rather do not make sense when they speak C. Parkinson's Disease - Parkinson's patients may have difficulty speaking loudly and clearly, but generally have trouble initiating and terminating locomotive movements D. Dysarthria - Patients affected by this may have trouble speaking, but will also have trouble eating, swallowing, and will have difficulty with fine motor control of their tongue E. Apraxia - Patients with apraxia have difficulty with motor planning, not speaking

You are treating in the spinal cord unit of a rehabilitation hospital and are working on transfers with your 21 year-old patient, Mr. Risk-Taker. He is able to abduct his arms, flex his shoulders, and extend his wrists (5/5 strength tested during manual muscle testing) but cannot extend his elbows (0/5 strength tested during manual muscle testing). You find that he has normal reflexes of the biceps and brachioradialis, but abnormal extensor plantar reflexes. Identify the level of his injury and appearance of the involved segment: A. C7 B. C5 C. T2 D. C6 E. C7

A. C7. Level is correct. Image is of the thoracic spine. B. C5. Patient would not be able to abduct or flex shoulders. Image is correct. C. T2. Patient would be able to extend elbows. Image is of thoracic spine. D. C6. Patient would have abnormal reflexes of brachioradialis and would not be able to extend wrists. Image is of thoracic spine. E. C7- CORRECT- Correct level and image

You are working in a free clinic in the suburbs of Who-Ville. Your receptionist, the Cat in the Hat, informs you that you have a double-evaluation coming from the office of the renowned Dr. Seuss. Thing 1 and Thing 2 will be seeing you for evaluation of their gait disturbances. They look similar at first glance: both demonstrate a wide based, staggering gaits, and neither has trouble initiating gait, but upon closer examination, you notice some subtle differences. Thing 1 double taps his foot on the floor before his foot slaps down, and he does not take his eyes off the ground while he ambulates. Thing 2's feet do not double tap, but rather, seem to be shaking as they reach for the ground in front of him. You also notice that Thing 2's eyes 'bounce' as he looks around the room and occasionally at the ground while he is ambulating. You suspect that Thing 1 is affected by ________, while Thing 2 is affected by ___________. a. Cerebellar ataxia; Athetosis b. Cerebellar ataxia; Sensory ataxia c. Sensory ataxia; Parkinson's disease d. Sensory ataxia; Cerebellar ataxia e. Sensory ataxia; Athetosis

A. Cerebellar ataxia; Athetosis - Thing 1 is not affected by cerebellar ataxia as he is able to double tap his foot on the ground, demonstrates no dysmetria, and has no difficulty with his eye movements. Furthermore, Thing 2 is not affected by athetosis, as this is characterized by writhing movements B. Cerebellar ataxia; Sensory ataxia - Thing 1 is not affected by cerebellar ataxia as he is able to double tap his foot on the ground, demonstrates no dysmetria, and has no difficulty with his eye movements. Thing 2 is not affected by sensory ataxia as this is characterized by patients staring at the ground in front of them while they walk. C. Sensory ataxia; Parkinson's disease - Thing 1 is likely affected by sensory ataxia as his feet slap the floor during gait and he must look at the ground while he walks, however, Thing 2 is likely not affected by Parkinson's disease as he does not have trouble initiating movement. D. Sensory ataxia; Cerebellar ataxia - CORRECT - Thing 1 is likely affected by sensory ataxia as his feet slap the floor during gait and he must look at the ground while he walks, and Thing 2 is likely affected by cerebellar ataxia as he demonstrates nystagmus and dysmetria when his feet near the ground. E. Sensory ataxia; Athetosis - Thing 1 is likely affected by sensory ataxia as his feet slap the floor during gait and he must look at the ground while he walks, however, Thing 2 is likely not affected by athetosis as this is characterized by writhing movements.

You are covering for another PT at your outpatient clinic and see his patient, 67 y/o Mrs. Jones. You notice she has a hard time finding a route to get around the clinic tables and machines. While taking her history, she tells you that she was spending time drawing geometric shapes with her 7 - year-old grandson. She is unable to re-draw the shapes her grandson drew. You suspect that she may have ______. If this is true, what else would she have problems doing? a. Construction apraxia; Organizing parts of a complex structure b. Ideational apraxia; Impaired knowledge of what action is associated with a particular object. c. Dressing/grooming apraxia; Difficulty getting dressed d. Sensory ataxia; loss of position sense in feet + legs e. Cerebellar Ataxia; Exaggerated difficulty on turns

A. Construction apraxia; Organizing parts of a complex structure - CORRECT- Patients with constructional apraxia have difficulty with route finding, reproducing geometric shapes, and organizing parts of a complex object B. Ideational apraxia; Impaired knowledge of what action is associated with a particular object.- A person with ideational apraxia has impaired knowledge of what action is associated with a particular object rather than difficulty organizing parts of a complex object. C. Dressing/grooming apraxia; Difficulty getting dressed- A person with dressing/grooming apraxia would struggle with getting dressed. D. Sensory ataxia; loss of position sense in feet + legs- A person with sensory ataxia would have loss of position sense in the feet + legs. Unsteady, wide based, double tap as heels then toes slap the ground and watch the ground for guidance. E. Cerebellar Ataxia, Exaggerated difficulty on turns-A person with Cerebellar ataxia has staggering, wide based, unsteady gait and exaggerated difficulty on turns.

During your acute care affiliation, you are working with Mr. Anders, a 22-year-old male who sustained head trauma after falling while skateboarding without a helmet. He has noted difficulty judging the distance of objects around him and bumps into objects frequently. You would like to test his vision. What results might you expect? A. Contralateral homonymous hemianopsia, suggesting damage to an optic tract. B. Unilateral blindness, suggesting damage to an optic nerve. C. Contralateral lower quadrant blindness, suggesting damage to the parietal lobe. D. Contralateral upper quadrant blindness, suggesting damage to a temporal lobe. E. Contralateral homonymous hemianopsia with macular sparing, suggesting occipital lobe damage.

A. Contralateral homonymous hemianopsia, suggesting damage to an optic tract. -The patient would have trouble seeing things in one of his visual fields, but should still be able to judge distances well. B. Unilateral blindness, suggesting damage to an optic nerve. CORRECT - The patient is demonstrating impaired depth perception, which would result from unilateral blindness. (Fred: include the concept of monocular and binocular vision and how that's related to depth perception.) C. Contralateral lower quadrant blindness, suggesting damage to the parietal lobe. -The patient would have trouble seeing things in the lower corner of one of his visual fields, but should still be able to judge distances well. D. Contralateral upper quadrant blindness, suggesting damage to a temporal lobe. -The patient would have trouble seeing things in the upper corner of one of his visual fields, but should still be able to judge distances well. E. Contralateral homonymous hemianopsia with macular sparing, suggesting occipital lobe damage. - The patient would have trouble seeing things in one of his visual fields, but should still be able to judge distances well.

You have been treating Mr. Biffle for his SCI at the NOA clinic, and you notice one day that he has burn marks all over his hands. Upon asking Mr. Biffle about the marks, he states that he doesn't know how or why they are there. It is wintertime and he notes that he does enjoy sitting near his fireplace for warmth in the evening. Damage to which tract is most likely causing Mr. Biffle to not know how these injuries occurred? A. Corticospinal Tract B. Dorsal column lesions C. Lateral column damage D. Spinothalamic Tract E. Mr. Biffle needs to just stop drinking and these injuries will stop

A. Corticospinal Tract: this would result in weakness, unsteadiness, stiffness in walking, increased tone and reflexes, extensor plantar responses and hyperactivity of stretch reflexes below a cord transection lesion which is due to the loss of descending modulatory influences. B. Dorsal column lesions: this would result in ataxia and clumsiness in the hands, loss of joint position and vibration senses C. Lateral column damage: this would result in altered sphincter function D. Spinothalmic Tract- CORRECT- Damage to this tract results in lost or altered cutaneous sensation or pain, painless injuries and burns due to lack of sensation E. Mr. Biffle needs to just stop drinking and these injuries will stop: INCORRECT: injuries are related to his spinal cord damage and not his drinking habits which he has stopped.

Mr. Smith is a 60-year-old carpenter who is attending your clinic for treatment of his Alzheimer's disease. You notice that he frequently puckers and licks his lips. While he is still able to successfully utilize a hammer and place nails, he is unable to pretend to do this with an imaginary hammer and nails. What disorder is Mr. Smith demonstrating? A. Dressing apraxia B. Construction apraxia C. Ideomotor apraxia D. Ideational apraxia E. Limb-Kinetic apraxia

A. Dressing apraxia. - The patient does not have difficulty getting dressed. B. Construction apraxia. - The patient does not have difficulty reproducing geometric figures, route finding, or organizing parts of a complex object. C. Ideomotor apraxia. CORRECT - The patient does have difficulty miming the use of an imaginary object. D. Ideational apraxia. - The patient has difficulty performing a series of tasks; however, he is still able to complete the tasks while manipulating real objects. E. Limb-Kinetic apraxia. - The patient does not have difficulty moving one of his limbs and has not lost dexterity.

Mrs. Jones is a 60-year-old woman who you are treating in an acute care hospital. Her chief complaint is that her eyelid is drooping and she cannot see as well as she used to. Upon taking her subjective information, you observe that her right eye is slightly deviated downward and outward. You immediately suspect oculomotor nerve palsy and perform a pupillary reflex test. Her pupils constrict in response to light. Unfortunately, you did not have time to thoroughly perform a chart review before seeing your patient. If you were to thoroughly review her past medical history you might expect to find all of the following comorbidities EXCEPT: A. Hypertension B. Tumor C. Diabetes D. Syphilis E. Vasculitis

A. Hypertension: Hypertension is associated with pupil-sparing oculomotor nerve palsy. B. Tumor: CORRECT: Compressive lesions are associated with pupil-involvement oculomotor nerve palsy. C. Diabetes: Diabetes is associated with pupil-sparing oculomotor nerve palsy. D. Syphilis: Syphilis is associated with pupil-sparing oculomotor nerve palsy. E. Vasculitis: Vasculitis is associated with pupil-sparing oculomotor nerve palsy. (Fred: the way the question is worded, all answers could be correct. Tumor is very general and could correspond to another kind of tumor which could present with oculomotor palsy... I won't go into too much detail, but you have to be specific about the compression on the nerve.)

While reviewing the charts of the patients you will see today, you notice that the imaging report for one of your new patients, Mrs. Franklin, notes, "left central facial nerve lesion." What symptoms might you observe with this patient? A. Impaired ability to turn up the corner of her mouth and wrinkle her forehead on her left side. B. Impaired ability to turn up the corner of her mouth and wrinkle her forehead on her right side. C. Impaired ability to turn up the corner of her mouth on her left side. D. Impaired ability to turn up the corner of her mouth on her right side. E. Complete paralysis of both sides of her face.

A. Impaired ability to turn up the corner of her mouth and wrinkle her forehead on her left side. - This impairment is expected with a left peripheral facial nerve lesion, not a left central lesion. B. Impaired ability to turn up the corner of her mouth and wrinkle her forehead on her right side. - This impairment is expected with a right peripheral facial nerve lesion, not a left central lesion. C. Impaired ability to turn up the corner of her mouth on her left side. - This impairment is expected with a right central facial nerve lesion, not a left central lesion. D. Impaired ability to turn up the corner of her mouth on her right side. CORRECT - This pattern is typical for a left central facial nerve lesion. E. Complete paralysis of both sides of her face. - A left central facial nerve lesion should only elicit observable symptoms in the right side of the patient's face, not both sides of the face. Both facial nerves would have to have central lesions for this presentation.

Mr. Hercules is a 66-year-old male on the neurological floor of an acute care hospital who was diagnosed with a left pontine lesion. Thinking back to your neuroanatomy, you recall that cranial nerves V through VIII are associated with the pons, but you cannot quite remember how a patient with a pontine lesion might present. After performing a cranial nerve examination it is quite clear that pontine lesions cause ________ A. Ipsilateral lateral rectus palsy and peripheral/lower motor neuron facial nerve palsy B. Contralateral lateral rectus palsy and peripheral/lower motor neuron facial nerve palsy C. Decreased visual acuity D. Blindness in the opposite side of both visual fields E. Decreased ability to smell

A. Ipsilateral lateral rectus palsy and peripheral/lower motor neuron facial nerve palsy - CORRECT; CN VI and VII are impacted by a pontine lesion. B. Contralateral lateral rectus impairment and peripheral/lower motor neuron facial nerve palsy- Pontine lesions are associated with ipsilateral rectus impairment, not contralateral impairment C. Decreased visual acuity - Decreased visual acuity is associated with optic nerve damage. Cranial nerve II is not associated with the pontine lesions. D. Blindness in the opposite side of both visual fields- This impairment is associated with optic tract/one lateral geniculate body damage, not pontine lesion. E. Decreased ability to smell - Associated with olfactory nerve damage, not pontine lesion.

You are working in a local acute-care hospital, treating 55 y/o Mr. Jenkins who was admitted after having a stroke. During the initial evaluation, he is able to understand and respond to all of your questions. However, despite his ordeal, you notice that he does not seem to have good safety awareness or fear of falling. His family also happens to mention that, since his stroke, he's been overeating. You suspect the stroke affected his _______, therefore causing ___________. a. L temporal lobe; Wernicke's aphasia b. Pre-frontal lobe; Abulic State c. Frontal lobe; Broca's Aphasia d. Left Parietal lobe; Gerstmann's Syndrome e. Superior Temporal Gyrus; Kluver- Bucy syndrome

A. L temporal lobe; Wernicke's aphasia-A person with this condition would have difficulty understanding what's being said to him. B. Pre-frontal lobe; Abulic State-A person with this condition would be awake but not speaking or interacting or it would be delayed. C. Frontal lobe; Broca's Aphasia-A person with this condition would have difficulty speaking. D. Left Parietal lobe; Gerstmann's Syndrome-A person with this would have dysgraphia, dyscalculia, finger agnosia, and left-right disorientation. E. Superior Temporal Gyrus; Kluver- Bucy syndrome-CORRECT- Damage to the superior temporal gyrus causes Kluver-bucy syndrome which leads to lack of fear, disinhibited eating and sexual behaviors.

You will be evaluating 30 year old Mr. Black, who has been diagnosed with Brown Sequard Syndrome at your inpatient rehab center today. His spinal cord has been severed on the right side by a bullet wound (hemisection in spinal cord). You know that, as a result of the organization of ascending and descending pathways within the spinal cord, he should demonstrate unilateral sensory and motor deficits and you plan to assess this during your evaluation as part of your differential diagnosis. What distribution of sensory and motor deficits would you expect this patient to have? A. Pain, temperature, discriminative touch, and proprioceptive deficits on the right; Motor deficits on the left. B. Motor deficits on the right; Pain, temperature, discriminative touch, and proprioceptive deficits on the left. C. Pain, temperature, and motor deficits on the right; Discriminative touch and proprioceptive deficits on the left. D. Discriminative touch, proprioceptive, and motor deficits on the right; Pain and temperature deficits on the left. E. Motor, discriminative touch, proprioceptive, pain, and temperature deficits on the right.

A. Pain, temperature, discriminative touch, and proprioceptive deficits on the right; Motor deficits on the left. - Discriminative touch and proprioception would be involved on the right, as they travel ipsilaterally in the dorsal columns. Motor would be involved on the right, as it also travels ipsilaterally in the corticospinal tract. Pain and temperature would be involved on the left, as they travel contralaterally in the spinothalamic tract. B. Motor deficits on the right; Pain, temperature, discriminative touch, and proprioceptive deficits on the left. - The motor deficits and pain and temperature deficits are correct. Discriminative touch and proprioception would be involved on the right, as they travel ipsilaterally in the dorsal columns. C. Pain, temperature, and motor deficits on the right; Discriminative touch and proprioceptive deficits on the left. - While the motor deficits are correct on the right, discriminative touch and proprioception would be involved on the right, as they travel ipsilaterally in the dorsal columns. Pain and temperature would be involved on the left, as they travel contralaterally in the spinothalamic tract. D. Discriminative touch, proprioceptive, and motor deficits on the right; Pain and temperature deficits on the left. -CORRECT - the spinothalamic, corticospinal, and dorsal column tracts would be impaired in this distribution below the level of the lesion. E. Motor, discriminative touch, proprioceptive, pain, and temperature deficits on the right. - Pain and temperature deficits should be present on the left, as they travel contralaterally in the spinal cord.

While treating in an inpatient clinic a 5 y/o boy named Scott walks in using Lofstrand crutches. He demonstrates truncal ataxia and is unable to sit up, stand, or walk without support. His mother reports he has been diagnosed with medulloblastoma. Which area of the brain does this disease affect? a) Paleocerebellum b) Neocerebellum c) Archicerebellum d) Occipital lobe e) Temporal lobe

A. Paleocerebellum- coordinates synergistic extremity movements for walking and running B. Neocerebellum- enables thumb opposition and grasping C. Archicerebellum-CORRECT controls head and neck movements D. Occipital lobe- this contains primary visual cortex E. Temporal lobe- this contains auditory perception

Patients A and B are both SCI patients on your schedule; however, they present with different signs and symptoms. Patient A exhibits diminished pain sensation in his feet and legs. He is currently seeing you for wound care treatment of an ulcer that developed after a small cut on his foot became infected without him noticing. Patent B demonstrates hyperactive stretch reflexes and general weakness and unsteadiness while walking. What tracts were affected in Patients A and B, respectively? A. Patient A: Spinothalamic tract; Patient B: Corticospinal tract B. Patient A: Corticospinal tract; Patient B: Dorsal column tracts C. Patient A: Spinothalamic tract; Patient B: Dorsal column tracts D. Patient A: Dorsal column tracts; Patient B: Corticospinal tract E. Patient A: Rubrospinal tract; Patient B: Corticospinal tract

A. Patient A: Spinothalamic tract; Patient B: Corticospinal tract - CORRECT - The spinothalamic tract directs pain signals from the body to the brain, leading to Patient A's symptoms. The corticospinal tract directs inhibitory motor signals to the lower motor neurons, leading to Patient B's motor symptoms. B. Patient A: Corticospinal tract; Patient B: Dorsal column tracts - Patient A's sensory deficits are related to the spinothalamic tract, not the corticospinal tract. Patient B would be expected to have proprioceptive or fine sensory deficits if he had a dorsal column injury. C. Patient A: Spinothalamic tract; Patient B: Dorsal column tracts - Patient A does have a spinothalamic tract injury; however, Patient B would be expected to have proprioceptive or fine sensory deficits if he had a dorsal column injury. D. Patient A: Dorsal column tracts; Patient B: Corticospinal tract - Patient A would be expected to have proprioceptive or fine sensory deficits if he had a dorsal column injury. Patient B does have a corticospinal tract injury. E. Patient A: Tectospinal tract; Patient B: Corticospinal tract - Patient A would be expected to have difficulty coordinating head turning to see objects or respond to visual or auditory stimuli. Patient B does have a corticospinal injury.

You are treating 50 y/o Mrs. Bright for the first time in an outpatient clinic. You notice that as you are walking toward her, her right arm is moving uncontrollably. As she reaches toward you to shake your hand, her uncontrollable movement disappears and she is able to complete the handshake without difficulty. You think she may have a _________. a) Postural tremor b) Resting tremor c) Intention tremor d) Dysmetria e) Nothing she's just super excited to start PT

A. Postural tremor-Appears when the affected part is actively maintaining a posture B. Resting tremor-CORRECT prominent at rest and decrease with voluntary movement C. Intention tremor -worsens as body part gets closer to target D. Dysmetria-This is an intention tremor that appears at the end of a movement E. Nothing she's just super excited to start PT- This is a resting tremor

Mr. Grotto Piazza is a 57-year-old Italian man whom you are treating at your outpatient PT clinic. When his wife brings him into PT the next day, she tells to you that her husband had trouble recognizing some of his family members at a family party this past weekend. "He recognizes all everyone's voices, but could not visually distinguish between his two brothers across the room." What neurological condition is associated with these symptoms and what area of the brain was injured with this condition? A. Prosopagnosia and the Occipital Lobe B. Agraphasthesia and the Occipital Lobe C. Prosopagnosia and the Temporal Lobe D. Asteriognosis and the Parietal Lobe E. Atherognosis and the Temporal Lobe

A. Prosopagnosia and the occipital Lobe- CORRECT: Prosopagnosia is the acquired loss of the ability to recognize faces that occurs with an insult to the occipital lobe (specifically the dissociation of the occipital cortex and the language cortex) B. Agraphasthesia and the occipital lobe: While the occipital lobe is affected with the condition in question, agraphasthesia is the inability to recognize numbers or letters written on the palm of the hand, not the inability to recognize faces C. Prosopagnosia and the temporal lobe: Prosopagnosia is the correct condition but occurs with an insult to the occipital lobe, not the temporal lobe. D. Astereognosis and the parietal lobe- While astereognosis occurs with an injury to the parietal lobe, astereognosis is the inability to distinguish/recognize small objects based on size, shape + texture when placed in a hand that has normal primary tactile sensory input, not the inability to recognize faces E. Atherognosis and the temporal lobe- atherognosis is not a type of agnosia and insult of the temporal lobe would cause auditory agnosias, not visual agnosias such as Prosopagnosia.

Mr. Jones, a 58-year-old male, has come to your outpatient physical therapy clinic for gait retraining 6 weeks status post left ankle fracture. Mr. Jones's fracture was secondary to a fall in his apartment; however, he cannot quite recall exactly what happened. His daughter has told you privately that her father has a drinking problem. Given his history of drinking and severe memory impairments, you suspect he has Korsakoff's syndrome. His daughter reports that her father has had multiple falls in the past six months and you also note that Mr. Jones has severe truncal ataxia and a wide based gait. Mr. Jones most likely has a lesion in which area of his central nervous system? (Choose the BEST answer). A. Right Parietal Lobe B. Vermis of the cerebellum C. Lateral Cerbellar Cortex D. Flocculonodular Lobe E. Left Medial Occipital Lobe

A. Right Parietal Lobe: Lesions in this area are associated with neglect of the left side. B. Vermis of the cerebellum: CORRECT: Alcohol preferentially poisons the vermis which is responsible for muscle tone for posture and locomotion, causing a characteristic syndrome of gait ataxia with sparing of the limbs. C. Lateral Cerbellar Cortex: Coordination of the ipsilateral arm and leg is typically more affected than trunk. D. Flocculonodular Lobe: Associated with vertigo. E. Left Medial Occipital Lobe: Lesions here are associated with visual impairment.

You are seeing Mr. Taco, a 22-year-old student at the University of Delaware, for anterior knee pain. Upon arriving at the clinic on Monday morning, the patient reports that he does not really remember what happened over the weekend, but has had difficulty seeing the ceiling to his left since he woke up on Sunday morning. Damage to this which lobe is associated with this visual field abnormality? A. Right Temporal Lobe B. Left Occipital Lobe C. Right Parietal Lobe D. Left Temporal Lobe E. Left Parietal Lobe

A. Right Temporal Lobe - This lesion is associated Meyer's Loop and with blindness in the upper quadrant. B. Left Occipital Lobe -This lesion is associated with contralateral hemifield blindness. C. Right Parietal Lobe - CORRECT; this area is associated with Meyers loop and results damage results in contralateral lower quadrant blindness. D. Left Temporal Lobe - with this lesion blindness would occur in the upper quadrant, not the lower quadrant, and the lesion would be on the right E. Left Parietal Lobe - this lesion is associated with contralateral lower quadrant blindness, but the lesion would be on the right since blindness is occurring on the left

You are seeing Mr. Taco, a 22-year-old student at the University of Delaware, for anterior knee pain. Upon arriving at the clinic on Monday morning, the patient reports that he does not really remember what happened over the weekend, but has had difficulty seeing objects on the ceiling to his left since he woke up on Sunday morning. Damage to this which lobe is associated with this visual field abnormality? A. Right Temporal Lobe B. Left Occipital Lobe C. Right Parietal Lobe D. Left Temporal Lobe E. Left Parietal Lobe

A. Right Temporal Lobe - This lesion is associated with blindness in the upper quadrant, not the lower quadrant. B. Left Occipital Lobe -This lesion is associated with contralateral hemifield blindness. C. Right Parietal Lobe - CORRECT; this area is associated with Meyers loop and results damage results in contralateral upper quadrant blindness. D. Left Temporal Lobe - with this lesion blindness would occur in the upper quadrant, not the lower quadrant, and the lesion would be on the right E. Left Parietal Lobe - this lesion is associated with contralateral lower quadrant blindness, but the lesion would be on the right since blindness is occurring on the left

While doing an initial evaluation in your outpatient clinic, you ask your 62 year old patient, Mrs. Houston, receives a call on her cell phone and, after excusing herself, talks to her daughter briefly, then hangs up and turns her attention back to you. A few minutes later, when you ask her to pretend to answer a call, without actually using her phone, she has no idea how to perform the task. Similarly, when you ask her to put on lipstick, she takes hers out and puts it on with no problem, but when you later ask her to pretend to put it on, she can't do it. You excuse yourself from the room and consult your colleague, Dr. Sheldon Cooper-Henderson, DPT, PhD, who immediately identifies her as having _______ since she can perform a learned task with real objects, but cannot pantomime or pretend to perform the same tasks, with imaginary objects. a. Sensory Ataxia b. Aphasia c. Cerebellar Ataxia d. Ideomotor Apraxia e. Ideational Apraxia

A. Sensory Ataxia -Sensory ataxia refers to the loss of position sense in the legs and feet B. Aphasia - Aphasia refers to difficulty with either understanding or forming speech C. Cerebellar Ataxia - Cerebellar ataxia refers to the inability of the cerebellum to provide appropriate corrections to movement, which generally results in wide based, staggering gait D. Ideomotor Apraxia - CORRECT - Ideomotor apraxia is the inability to pantomime the use of an imaginary object E. Ideational Apraxia - Ideational apraxia is impaired knowledge of what action is associated with a particular object

You are working in an outpatient physical therapy clinic in downtown Baltimore and are performing an initial evaluation with Mr. Bubbles a 38-year-old male. While taking your subjective, you learn that Mr. Bubbles' is a confidential informant for the Baltimore City Police and he uses the money he earns to buy drugs. He has come in with a chief complaint of back pain and you also notice that he ambulates with decreased hip flexion. Upon further questioning, Mr. Bubbles tells you that he walks funny because walking and stair climbing hurt his groin. Mr. Bubbles is sweating during the evaluation, he is also running a fever of 102 degrees Fahrenheit, and he informs you that he has lost 15 pounds recently, "without even trying." He suspects all these odd symptoms are secondary to his diagnosis of HIV one year ago. You find that he has normal sensation in his groin. Your next step is to: A. Start treating Mr. Bubbles for extensor muscle spasms and hip flexor tendinopathy next visit when he gets the HIV symptoms under control. B. Immediately refer as you suspect the patient has a paraspinal abscess. C. Immediately refer as you suspect the patient has a reticulospinal lesion. D. Immediately refer as you suspect dorsal column syndrome. E. Immediately refer as you suspect damage to the vestibulospinal tract.

A. Start treating Mr. Bubbles for extensor muscle spasm and hip flexor tendinopathy next visit when he gets the HIV symptoms under control. Although he may have extensor muscle spasm and hip flexor tendinopathy, you must rule out paraspinal infection. You cannot ignore the fever, sweating, and weight loss as these are all red flags. You are better than that. B. Immediately refer as you suspect the patient has a paraspinal abscess-CORRECT- Patient has all the classic signs of a paraspinal abscess: fever, weight loss, sweats, and pain to the iliopsoas. C. Immediately refer as you suspect the patient has a reticulospinal lesion. Reticulospinal lesion would impact patient's alertness. D. Immediately refer as you suspect dorsal column syndrome. You would expect loss of proprioception, vibration, and position sense. This syndrome is most commonly associated with syphilis, not HIV. E. Immediately refer as you suspect conus medullaris syndrome. Although this syndrome can be associated with back pain, you would also expect saddle anesthesia and early sphincter dysfunction.

One of your classmates arrives to class on Monday morning looking as though she thoroughly enjoyed her Sunday Funday. Although her memory and current appearance leave much to be desired, she reports that she thinks she hit her head against the floor at Klondike Kate's when performing a complicated dance move. You notice that she must tilt her head to the left in order to see the projector. Which nerve do you suspect was damaged during the fall on Sunday Funday? A. Trigeminal Nerve B. Abducens Nerve C. Optic Nerve D. Oculomotor Nerve E. Trochlear Nerve

A. Trigeminal Nerve- Damage to this nerve is not associated with vision impairment on head tilt. B. Abducens Nerve- Damage to this nerve is associated with impaired visual tracking, but will not lead to a head tilt. C. Optic Nerve - Damage to this nerve is associated more with visual acuity and your classmate would have trouble reading the slides. D. Oculomotor Nerve - Damage to this nerve is associated with visual tracking but won't lead to a head tilt. E. Trochlear Nerve - CORRECT; Trochlear nerve damage associated with head trauma is associated with a head tilt. (Fred: vertical diplopia could be the reason for the head tilt.)

Mrs. Fastigius is a 67-year-old woman whom you are treating in a rehabilitation hospital after she was discharged from the acute care hospital yesterday. You have not received the imaging for this patient, but she was cleared for PT today. When you ask Mrs. Fastigius to pick up a piece of string on the table, she responds with an erratic tone and her left hand begins shaking on her way to pick up the string. Later during the physical exam you ask her to stand up and walk ten feet to the door and back. Upon standing, she takes a short pause to collect herself and then begins walking. You notice a severe lack of coordination of the left arm and leg as she walks to the door and back. A lesion to what important coordination center does this presentation represent? A. Vermis B. Lateral Cerebellar Cortex C. Flocculonodular Lobe D. Left Parietal Lobe E. Right Medial Occipital Cortex

A. Vermis- A lesions in this area is associated with truncal ataxia and sparing of the limbs. B. Lateral Cerebellar Cortex- CORRECT: A lesion of this area is associated with coordination deficits of the ipsilateral arm and leg (more affected than trunk); ataxic dysarthria (irregular speech cadence) C. Flocculonodular Lobe- A lesions of this area is associated with vertigo. D. Left Parietal Lobe- A lesion of this area associated with neglect of the right side. E. Right medial Occipital Cortex- A lesion in this region is associated with visual impairment.

You are treating in an acute care setting and you enter the room of 80 y/o Ms. Keri. During your exam you realize she has impaired calculation, right and left confusion, finger agnosia, and agraphia. You think she may have caused by lesions to the _______ lobe. a) Wernicke's aphasia; temporal b) Broca's aphasia; temporal c) Gerstmann's syndrome; parietal d) Gerstmann's syndrome; occipital e) Cortical syndrome; parietal

A. Wernicke's aphasia; temporal- Wernicke's causes receptive aphasia. B. Broca's aphasia; temporal - Broca's causes expressive aphasia. C. Gerstmann's syndrome; parietal CORRECT- Gerstmann's syndrome is characterized by these symptoms and is a lesion to dominant parietal lobe. D. Gerstmann's syndrome; occipital- Gerstmann's is the correct answer but the lesion is located in the parietal lobe not the occipital lobe. E. Cortical syndrome; parietal -Cortical syndrome causes apraxia or learned and skilled movements.

You are treating 65 y/o Ms. Jones in an outpatient clinic. During your subjective exam, you realize she has aphasia, which affects the lobe. You know this because, although you are speaking clearly and she can hear you, she cannot comprehend the words you are saying. a) Wernicke's; temporal b) Broca's; parietal occipital c) Conduction; posterior frontal d) Wernicke's; posterior frontal e) Broca's; temporal

A. Wernicke's; temporal -CORRECT damage to Wernicke's causes receptive aphasia and its located in the temporal lobe B. Broca's; parietal occipital - Damage to Broca's area causes expressive aphasia and damage to parietal occipital lobe would cause visual receptive defects C. Conduction; posterior frontal -Conduction is poor reading, naming, and repetition posterior frontal lobe damage would give you expressive writing aphasia D. Wernicke's; posterior frontal- Wernicke's is correct but posterior frontal lobe damage would give you expressive writing aphasia E. Broca's; temporal - Broca's area causes expressive aphasia

Mr. M is a stroke patient you are treating in the hospital. When you arrive to the room, the speech therapist is present. You know that about 23% of patients with strokes have aphasia, or difficulty speaking, and that the degree of recovery correlates with the initial severity. What is the most common time frame that you expect this patient to make the most gains in aphasia recovery? a. 1-3 months b. 1-3 weeks c. 3-6 months d. 6-9 months e. 3-6 weeks

a. 1-3 months - Incorrect. b. 1-3 weeks - Incorrect. c. 3-6 months - CORRECT - Continued recovery can occur after this, but this is the most common time frame. d. 6-9 months - Incorrect. e. 3-6 weeks- Incorrect.

Mr. George is a 21-year-old male who recently was involved in a traumatic motor vehicle collision. You are treating him in an acute care setting and given the traumatic nature of his accident, you decide to perform a full neurological exam. The neurological exam reveals a positive Babinski sign. Although a positive Babinski sign would not be alarming in the pediatrics population, you are concerned because a positive Babinski sign is abnormal after the age of ____. a. 2 months old b. 18 months old c. 24 months old d. 4 years old e. 16 years old

a. 2 months old- It is normal for children of this age to extend their toes during this test because their spinal tracts are not yet fully myelinated. b. 18 months old- It is normal for children of this age to extend their toes during this test because their spinal tracts are not yet fully myelinated. c. 24 months old- CORRECT- Spinal tract myelination is more fully matured. d. 4 years old - By this age, extended toes with Babinski testing would indicate pathology. e. 16 years old- By this age, extended toes with Babinski testing would indicate pathology.

During your evaluation of Mr. Zortman, a 54 year old male in your outpatient clinic, he reports having dizziness, confusion, loss of memory, decreased coordination and balance, numbness and weakness on his left side and difficulty understanding his wife. In order for this to be classified as a TIA, these symptoms must be resolved within a maximum of _____ hours. a. 24 b. 48 c. 36 d. 12 e. 72

a. 24- CORRECT- symptoms must be resolved in 24 hours, if longer it would be considered a stroke b. 48- symptoms lasting this long would be considered a stroke c. 36- symptoms lasting this long would be considered a stroke d. 12- the symptoms don't need to be resolved in 12 hours e. 72- symptoms lasting this long would be considered a stroke

While on your acute care affiliation, you do a chart review on Mr. Carmen, a 55 y/o AA man, for a PT initial eval. You notice that the patient was initially properly anticoagulated with an INR of ______. However, over the span of his month-long hospital stay you notice his INR has largely fluctuated over the 2 times it was checked. You are now concerned because _____. a. 3.5, his INR of 3.5 was too low b. 2.5, he likely is having a hemorrhagic stroke c. 1.0, his INR of 1.0 was too high d. 2.3, his INR was not checked often enough e. 3.0, you are not concerned because you are only a PT and the doctors surely know what they are doing

a. 3.5, his INR of 3.5 was too low- the proper therapeutic dose is between 2 and 3 so an INR of 3.5 is too high b. 2.5, he likely is having a hemorrhagic stroke- an INR of 2.5 is appropriate, so deciding he is having a hemorrhagic stroke is not appropriate (Fred: If he is having a hemorrhagic stroke (or hemorrhage of any kind) an INR of 2.5 can be at minimal dangerous and possibly catastrophic for the patient) c. 1.0, his INR of 1.0 was too high- the proper therapeutic dose is between 2 and 3 so an INR of 1.0 is not too high d. 2.3, his INR was not checked often enough- CORRECT- 2.3 is an appropriate INR, but checking INR levels should be checked at minimum once per week (Fred: If he started at 2.3 and went to >3, you'd be concerned. I realize you said month long hospital stay which implies 4 checks, however, 3-4 in one month is acceptable) e. 3.8, you are not concerned because you are only a PT and the doctors surely know what they are doing- an INR of 3.8 is too high, and recognizing this makes it is your responsibility

The outpatient PT clinic that smartly hired you out of school is very progressive and has a number of clients that come in for preventative purposes. Your caseload for the day has people with whom you want to discuss stroke prevention. Who is most likely to benefit from your advice? a. 85-year-old female b. 60-year-old male who has had a previous stroke c. 48-year-old with a family history with both parents having had strokes d. 55-year-old s/p knee replacement currently taking warfarin with an INR of 4-5. e. 90-year-old male with a history of previous stroke

a. 85-year-old female - age and sex are unmodifiable risks b. 60-year-old male who has had a previous stroke - previous stroke is an unmodifiable risk c. 48-year-old with family history with both parents having had strokes- family history is an unmodifiable risk d. 55-year-old s/p knee replacement currently taking warfarin with an INR of 4-5 -CORRECT - drug dose can be modified. Warfarin is at a therapeutic dose when at an INR of 2-3. e. 90-year-old male with a history of previous stroke- all are unmodifiable risk factors

Mr. Smith, an 18 year old Caucasian male, presents on your caseload in Acute Care following a MVA. Your chart review reveals the patient has a GCS of 13 and has demonstrated verbal aggression towards staff and family members. Upon examination, the patient lacks ability to maintain focus while you are taking his history and demonstrates impulsivity as he jumps out of bed to go to the bathroom, knocking over his IV. As the patient attempts to drag his IV to the bathroom, you notice he demonstrates gait disturbance resembling cerebellar ataxia. Mr. Smith's mother is present in the room and questions why her son has suddenly developed such aggressive and impulsive behavior along with "a funny walk", despite the normal CT. You explain to the patient's mother: a. A CT is not able to adequately identify the severity of the TBI as defined by a GCS of 13 b. The patient is demonstrating these behaviors due to the steroids provided on arrival to the trauma center. c.The CT was not an appropriate test. The patient's mother should request he receive a standard radiograph to truly determine the severity of his brain injury. d. The patient has diffuse axonal injury with damage spread throughout multiple areas of the brain, not detected by CT. e. The patient is obviously crying out for attention with his behaviors and will get over it by tomorrow if she ignores it.

a. A CT is not able to adequately identify the severity of the TBI as defined by a GCS of 13.-INCORRECT- Although the extent of the TBI cannot always be fully visualized with conventional neuroimaging, the patient does not have a severe TBI according to the GCS score of 13. A GCS score of 13-15 by definition is considered mild. b. The patient is demonstrating these behaviors due to the steroids provided on arrival.-INCORRECT- Steroid use is not indicated in the acute TBI patient due to increased mortality and disability. c. The CT was not an appropriate test. The patient's mother should request he receive a standard radiograph to truly determine the severity of his brain injury. INCORRECT- While an MRI may be more appropriate to assess DIA compared to a CT, a standard radiograph will not provide additional information regarding brain tissue damage. A standard radiograph will only determine bony abnormality. d. The patient has diffuse axonal injury with damage spread throughout multiple areas of the brain, not detected by CT. -CORRECT- DAI results from shearing of axons along the gray and white matter junctions. The patient demonstrates impulsivity and aggressiveness typical of frontal and temporal lobe damage with head injury as incurred during an MVA. Cerebellar ataxia is also due to DIA often more pronounced along the cerebellar peduncles. e. The patient is obviously crying out for attention with his behaviors and will get over it by tomorrow if she ignores it. -INCORRECT- It is extremely unlikely the patient will overcome the current deficits in executive function within a day, as deficits may persist for years and may worsen with time.

You are working at Magee Rehab Hospital with a SCI patient whose PVR (post-void residual) is 500mL. As an educated PT from UD, you know that a PVR > 400mL in a SCI patient can cause a vesicoureteral reflux. All of the following are possible characteristics of the vesicoureteral reflux (VUR) except: a. Abnormal flow of urine from bladder into ureters b. Hypertension c. Possible pyelonephritis d. Gall stones e. UTI

a. Abnormal flow of urine from bladder into ureters - This is the definition of VUR b. Hypertension - Some individuals with VUR are at an increased risk for HTN c. Possible pyelonephritis - Urine traveling from bladder all the way up to kidneys can cause pyelonephritis d. Gall stones - CORRECT Gall stones are not correlated with VUR e. Urinary Tract Infection (UTI) - VUR can lead to a UTI

Your patient, Jiminy Cricket, checked himself into the hospital losing vision in his left eye following a vigorous workout. His vision has since returned, but the attending physician has ordered an ABCD2 evaluation. You know that the ABCD2 is a predictor of ischemic stroke, but cannot remember what all of the letters stand for. Which of the following is not part of the ABCD2 score? a. Age b. Blood pressure c. Carotid stenosis >25% <-correct d. Duration of TIA e. Presence of Diabetes

a. Age INCORRECT. Age is a part of the ABCD score. b. Blood pressure INCORRECT. Blood pressure is a part of ABCD score. c. Carotid stenosis >25% CORRECT. C in ABCD is for clinical features, meaning their signs/symptoms. d. Duration of TIA INCORRECT. D is for duration of TIA. e. Presence of Diabetes INCORRECT. Other D is for presence of Diabetes.

Mr. Smith, a 48 year old Hispanic male, presents to outpatient PT for an initial evaluation reporting episodes of vertigo. The patient reports he was previously treated at another therapy clinic for BPPV and the treatment had provided some relief, however, the episodes have worsened over the past weeks. While taking his history the patient reports having some difficulty completing the Dizziness Handicap Index because the words are blurred. Upper and Lower Extremity sensory testing reveals sensory loss of the right UE and LE. Which of the following vascular pathologies should be considered in your differential diagnosis? a. Anterior Cerebral Artery CVA b. Lacunar Stroke of the Internal Capsule c. Mid-Basilar Artery Stroke d. Lacunar Stroke of the Upper Brainstem e. Peripheral Vascular Disease

a. Anterior Cerebral Artery CVA- Although symptoms of ACA are typically of contralateral weakness, sensory loss, gait apraxia, and disinhibition they typically do not include dysphagia. b. Lacunar Stroke of the Internal Capsule - A patient suffering from a Lacunar stroke within the Internal Capsule may present with contralateral hemiparesis and hemisensory loss, however will not present with visual loss. c. Mid-Basilar Artery CVA- A mid-basilar artery stroke will typically present with quadriparesis rather than contralateral loss. d. Lacunar Stroke of the Upper Brainstem: CORRECT- Lacunar strokes of the upper brainstem/cerebellum can result in not only Vertigo, but also diplopia and dysarthria. Horner's syndrome is a common syndrome associated with Lacunar strokes in this area with ipsilateral facial weakness, sensory loss and contralateral body sensory loss. e. Peripheral Vascular Disease- PVD refers to the obstruction of arteries from atherosclerosis or inflammatory processes. PVD may lead to stenosis, embolism or thrombus however commonly refers to peripheral vasculature rather than vasculature of the brain.

A 57 year old African American male, is added to your schedule in Acute Care with the diagnosis of an acute CVA. During the chart review, you come across these MRI images (see above). When you visit the room after lunch, you notice the patient has deviation of the tongue to the left, is hoarse and appears to be pocketing food in his mouth. Based on the images what artery do you believe to be involved? Which cranial nerves might you expect to be affected based on the limited patient presentation. a. Anterior Cerebral Artery; Hypoglossal Nerve, Vagus Nerve, Glossopharyngeal b. Middle Cerebral Artery; Vagus Nerve, Abducens Nerve, Hypoglossal Nerve c. Anterior Spinal Artery; Hypoglossal Nerve, Glossopharyngeal Nerve; Trochlear Nerve d. Basilar Artery; Hypoglossal Nerve, Vagus Nerve, Glossopharyngeal Nerve e. Axillary Artery; Hypoglossal Nerve, Vagus Nerve, Glossopharyngeal Nerve

a. Anterior Cerebral Artery; Hypoglossal Nerve, Accessory Nerve, Olfactory Nerve- The corresponding cranial nerves are correct for the patient presentation however, the Anterior Cerebral Artery is a branch of the Internal Carotid supplying the Frontal and Parietal Lobes and does not provide circulation to the Medulla where the corresponding cranial nerves arise. b. Middle Cerebral Artery; Vagus Nerve, Glossopharyngeal Nerve, Hypoglossal Nerve-The Middle Cerebral Artery arises from the Internal Carotid artery supplying the mid frontal and parietal lobes along with the superior temporal lobes. The Middle Cerebral artery does not supply the Medulla. The Abducens Nerve provides lateral visual tracking which was not noted as a current deficit. c. Anterior Spinal Artery; Hypoglossal Nerve, Glossopharyngeal Nerve; Trochlear-The Anterior Spinal Artery supplies the anterior portions of the spinal cord which can lead to bilateral effects on the corticospinal and spinothalamic tracts. The Trochlear Nerve is not involved as the patient did not present with deficits in downward gaze. d. Basilar Artery; Hypoglossal Nerve, Vagus Nerve, Glossopharyngeal- CORRECT- The Basilar artery is formed from the Vertebral Arteries and supplies branches to the Medulla, Pons and Cerebellum. Damage to the Hypoglossal nerve would be indicated with tongue deviation, while damage to the Vagus and Glossopharyngeal nerves may be indicated due to hoarseness and pocketing of food in the mouth. e. Axillary Artery; Hypoglossal Nerve, Vagus Nerve, Glossopharyngeal Nerve- The nerves indicated are correct, however the Axillary Artery supplies blood to the lateral thorax and axilla and is surrounded by the Brachial Plexus.

Ms. Eyre is a 17-year-old female being seen at an inpatient center. During a lower quarter screen, you observe generalized weakness and atrophy, reduced reflexes and involuntary muscle fasciculations of the quadriceps of which the patient is unaware. These signs are consistent with... a. Anterior horn cell pathology b. Fasciculus gracilis pathology c. Dorsal horn cell pathology d. Primary motor cortex infarct e. Pontine infarct

a. Anterior horn cell pathology - CORRECT - Damage to alpha motor neurons housed in the anterior horn of the spinal column will result in these symptoms associated with pathologies such as Polio and ALS b. Fasciculus gracilis pathology - Damage to this bundle of ascending tracts in the posterior column of the spinal cord would result in diminished proprioception of the lower extremity. c. Dorsal horn cell pathology - Damage to these cells would result in diminished light touch, proprioception and vibration sensation. d. Primary motor cortex infarct - This upper motor neuron lesion would result in heightened reflexes and spasticity e. Pontine infarct - Lesions to this area will result in facial hemiparesis, dysarthria, somatosensory disturbance and horizontal gaze abnormalities

Ms. Reynolds is a 16 year old caucasian female you are evaluating in the acute care setting one day status post ischemic stroke. You administer the National Institutes of Health Stroke Scale (NIHSS) to assess her rehab potential. All of the following were tested and used to assess her prognosis as part of the NIHSS except: a. Best gaze (follow finger) b. Motor (raise each arm, leg) c. Best visual (visual fields) d. Facial palsy (show teeth, raise eyebrows) e. Ataxia (gait analysis)

a. Best gaze (follow finger) - This is tested by NIHSS b. Motor (raise each arm, leg) - This is tested by NIHSS c. Best visual (visual fields) - This is tested by NIHSS d. Facial palsy (show teeth, raise eyebrows) - This is tested by NIHSS e. Ataxia (gait analysis) - CORRECT - Ataxia is not tested by the NIHSS

Your new patient is a 23 year old male who is status post motorcycle collision. You begin introducing yourself to him when you notice you've lost his attention and he's now paying attention to something outside the window. His mom is also in the room with you and she mentions he's been much more depressed, less motivated, but also more impulsive. His signs and symptoms are likely due to a lesion in which area? a. Anterior-inferior temporal contusion b. Frontal polar and orbital frontal c. Basal ganglia/internal capsule hemorrhage d. Superior mesial region of the frontal lobe e. Posterior cerebral artery infarct

a. Anterior-inferior temporal contusion- INCORRECT - a lesion located here would likely cause altered sexual behavior, agnosia, psychosis, temporal-spatial disorientation b. Frontal polar and orbital frontal- CORRECT - a lesion here would cause impulsivity, poor insight, inability to sustain attention, lack of motivation, and personality changes. c. Basal ganglia/internal capsule hemorrhage-INCORRECT - a lesion here would cause hemiparesis, incoordination, hypertonia, movement disorder, tremor, neglect, visual-perceptual disturbance, and aphasias d. Superior mesial region of the frontal lobe-INCORRECT - a lesion here would present with akinesia, apathy, and mutism. e. Posterior cerebral artery infarct- INCORRECT - a lesion here would cause hemianopia, cortical blindness, amnesia, aphasia, alexia, anemia, agraphia, neglect and visual-spatial disturbance

Mr. Jones, a 22 year old Caucasian male, is currently participating in inpatient physical therapy following a prolonged hospital stay. Mr. Jones was injured in a motorcycle collision resulting in an intracranial hemorrhage within the frontal lobe. After 3 weeks of therapy the patient has not improved in ability to perform simple tasks related to activities of daily living and requires guidance for dressing and feeding. Today in therapy Mr. Jones starts grabbing his skin and scratching himself, yelling "spiders, spiders everywhere!" You believe your patient may be having a subclinical seizure manifested as a hallucination. You recognize this subclinical seizure may predispose the patient to later development of epilepsy. The Primary Care Doctor tells you the patient was administered anti-convulsants within the first week post-TBI and therefore has no risk factors for developing seizures. What knowledge regarding post-TBI seizures should you discuss with the primary care physician? a. Anticonvulsants do not reduce the risk of late (>1 week post trauma) seizures and development of seizure after the first 24 hours may predict later development of epilepsy. b. Mr. Jones has an increased chance of developing epilepsy due to his young age. c.There is no information to discuss. The primary care physician is correct. Seizures typically will occur at least 2 years post TBI. d. There is no information to discuss. The primary care physician is correct. Anticonvulsants will reduce overall excitation in the brain for up to 2 years post administration preventing seizure. e.There is no information to discuss. The primary care physician is correct because he went to Medical School so you should leave it alone.

a. Anticonvulsants do not reduce the risk of late (>1 week post trauma) seizures and may predict later development of epilepsy. CORRECT- Anticonvulsants reduce the incidence of early post-traumatic seizures only. Also, a seizure after the first 24 hours may predict the later development of epilepsy in 70% of those with TBI. b. Mr. Jones has an increased chance of developing epilepsy due to his young age.- INCORRECT- Age is not a risk factor for epilepsy. Skull fractures, wounds penetrating the brain, history of chronic alcohol use, intracranial hemorrhage and increased severity of injury are all known risk factors for epilepsy. c. There is no information to discuss. The primary care physician is correct. Seizures typically will occur at least 2 years post TBI. - INCORRECT- In 75 to 80 percent of patients seizures will occur in the first 2 years after injury. d. There is no information to discuss. The primary care physician is correct. Anticonvulsants will reduce overall excitation in the brain for up to 2 years post administration preventing seizure. INCORRECT- Anticonvulsants do not reduce the risk of seizures after the first week post TBI. e. There is no information to discuss. The primary care physician is correct because he went to Medical School so you should leave it alone.-INCORRECT- The PT has an obligation to provide the physician with the evidence to support their conclusions in order to protect the health of the patient.

Mrs. Larson is a 63 years old patient who presents to acute care PT status post acute cerebral ischemia. This is your first time seeing her and you'd like to get a general sense of her potential to making a functional recovery. Which of the imaging choices would provide you with the most relevant information? a. Arteriography b. MRI c. Diffusion/perfusion weighted MRI d. CT e. Ultrasonography

a. Arteriography - good for detecting medium-large stenotic or occluded arteries in the head or neck b. MRI - good for providing information on subacute intracerebral hemorrhage c. Diffusion/perfusion weighted MRI - CORRECT - provides comparative measure of perfused versus non-perfused tissue regions (the extent of damage and salvageable tissue) (Fred: You can't really separate out MRI from diffusion/perfusion MRI. Keep in mind that no imaging modality has great correlation data with functional recovery.) d. CT - very good at detecting a hemorrhage e. Ultrasonography - good for detecting stenotic or occluded arteries in the neck and in some intracranial vessels

Mrs. McGrath is a 38 y/o female you are treating on your rehabilitation affiliation with a diagnosis of Syringomyelia. You recall, from your amazing neuroanatomy course, that this disorder is characterized by a fluid-filled cavity in the spinal cord that usually expands slowly over time. Which of the following is NOT associated with syringomyelia? a. altered CSF dynamics b. tumor c. trauma d. modulation of autonomic nervous system functions e. tremors

a. CSF dynamics -- CSF flow through the central canal is disrupted in syringomyelia. b. tumor -- syringomyelia may be a result of a tumor. c. trauma -- syringomyelia may be a result of trauma d. modulation of autonomic nervous system functions -- parasympathetic and sympathetic nervous systems may be disrupted by the syrinx. e. tremors -- CORRECT! Tremors are associated with basal ganglion lesions.

Edward Dunkin is a 68 year old male coming into your outpatient clinic for an initial evaluation. He is coming in via direct access for new onset of acute low back pain. You learn that his back pain just started last week. Your first action will be... a. Assess lumbar AROM b. Assess repeated Motions c. Send him to primary care physician to rule out prostate mets (this might be too much of a hint, maybe just refer to primary care physician) d. Assess straight leg raise test e. Send him to ER for a fracture

a. Assess lumbar AROM (incorrect - in a 65 y.o. male or older with new onset of back, MUST RULE OUT PROSTATE METS) b. Assess repeated Motions (incorrect - in a 65 y.o. male or older with new onset of back, MUST RULE OUT PROSTATE METS) c. Send him to primary care physician to rule out prostate mets (correct) d. Assess straight leg raise test (incorrect - in a 65 y.o. male or older with new onset of back, MUST RULE OUT PROSTATE METS) e. Send him to ER for fracture (incorrect - in a 65 y.o. male or older with new onset of back, MUST RULE OUT PROSTATE METS) You should mention that he has never had back pain before just to make it a slam dunk.

You are evaluating a 5-year-old girl with spina bifida who underwent a procedure to have the fluid filled-sac on her low back surgically closed approximately 6 months ago. Scarring from the surgery has resulted in a tethered cord. The following may be associated with a tethered cord, with the exception of: a. Asymmetrically decreased strength in the lower extremities b. Decreased respiratory function c. Lower extremity spasticity d. Early development of or increasing degree of scoliosis e. Fecal and/or urinary incontinence

a. Asymmetrically decreased strength in the lower extremities - consistent with lower spinal cord lesion. b. Decreased ventilatory function - CORRECT; this is not typically associated with a tethered cord since the phrenic nerve innervates the diaphragm, assists with ventilation, and typically emerges from C3-C5 and would not be affected by a spinal cord lesion in the lumbar region. c. Lower extremity spasticity - consistent with lower spinal cord lesion. d. Early development of or increasing degree of scoliosis - the patient may self-correct to decrease irritation on the spinal cord, which over time may lead to scoliosis. e. Fecal and/or urinary incontinence - may occur with lower spinal cord lesion.

While reviewing the radiology report for Mr. Sheppard, a 47 year old male, you discover he has stenosis in his left ophthalmic artery leading to a TIA. What symptom do you expect Mr. Sheppard to report? ("what symptom did he or might have reported before coming into the hospital... it's considered an emergency to be worked up immediately.") a. Ataxia b. Aphasia c. Loss of touch/temperature sensation in the involved limbs d. Transient monocular blindness (amaurosis fugax) e. Vertigo

a. Ataxia- this would result from a vertebrobasilar TIA b. Aphasia- this would result from stenosis in the MCA c. Loss of touch/temperature sensation in the involved limbs- this would result from stenosis in the MCA d. Transient monocular blindness (amaurosis fugax)- CORRECT- this would result from stenosis in the ophthalmic artery e. Vertigo- this would result from a vertebrobasilar TIA

Mr. Caffrey, 47 year old male, comes to Drayer Outpatient PT for treatment of his lateral ankle sprain. To challenge his balance, you have him perform SLS with his arms outstretched. While performing this exercise, you notice his wrists begin to flap. Patient also demonstrates confusion during treatment. Upon further investigation into his medical history, you notice he has cirrhosis of the liver secondary to alcohol abuse. What movement disorder associated with liver failure demonstrates wrist flap when arms are held outstretched? a. Ballismus b. Asterixis c. Dystonia d. Chorea e. Athetosis

a. Ballismus - flinging movements of a limb b. Asterixis - CORRECT - wrist flap when arms are held outstretched c. Dystonia - fixed, abnormal posture d. Chorea - brief, random, non-repetitive movements of fingers, extremities, face and trunk e. Athetosis - twisting and writhing movements of the extremities, trunk and sometimes face

Brady, a 3 year old boy, presents to your pediatric outpatient clinic with Torticollis. He demonstrates a sustained abnormal posture in L cervical sidebending and R cervical rotation. How would you classify this movement disorder? a. Ballismus b. Asterixis c. Dystonia d. Chorea e. Athetosis

a. Ballismus - flinging movements of a limb b. Asterixis - wrist flap when arms are held outstretched c. Dystonia - CORRECT - fixed, abnormal posture d. Chorea - brief, random, non-repetitive movements of fingers, extremities, face and trunk e. Athetosis - twisting and writhing movements of the extremities, trunk and sometimes face

Mrs. Lutz is a 66 y/o female you are treating in the NOA clinic for vestibular dysfunction. During cranial nerve testing, Mrs. Lutz demonstrates a vertical jerk nystagmus. In your differential diagnosis, in addition to a peripheral nerve lesion of CN VIII, you will want to include _________. a. Brainstem dysfunction b. Meyers loop pathology c. Broca's area pathology d. Basal ganglia dysfunction e. Prefrontal cortical pathology

a. Brainstem dysfunction-CORRECT- In addition to a peripheral nerve lesion, vertical nystagmus could also occur as a result of a central lesion. b. Meyers loop pathology - would result in visual impairments c. Broca's area pathology - would result in vocal impairments. d. Basal ganglia dysfunction - would result in resting tremor, but would not result in nystagmus e. Prefrontal cortical pathology - would result in personality changes and difficulties with complex motor task planning.

Mrs. Carmen San Diego is a 29 y.o. female with a T7 SCI being seen in rehab. She just returned from a luxurious trip to the countryside of Ireland. She showed you pictures of the cows on the farm on which she stayed and told you how good their unprocessed milk was! Today she has cancelled her appointment because she has a fever, sweats and flu-like symptoms. Based on her recent history, what may be the cause for these new onset symptoms? a. Brucellosis b. Myelopathy c. Autonomic Dysreflexia d. Lupus e. Saddle Anesthesia

a. Brucellosis (CORRECT - brucellosis is a subacute syndrome that can occur when unpasteurized milk is consumed) b. Myelopathy (incorrect - myelopathy can occur chronically in patients infected in syphilis, HIV, or HTLV-I) c. Autonomic Dysreflexia (incorrect - autonomic dysreflexia is a condition caused by a noxious stimulus resulting in increased BP, decreased HR, anxiety, flushed appearance) d. Lupus (incorrect - this is an autoimmune disease) e. Saddle Anesthesia (incorrect - this is symptom of conus medullaris syndrome)

Mr. Jones is a 45 year old patient with diabetes who presents to your outpatient PT clinic with complaints of an inability to blink his R eye. Upon evaluation you observe an inability of the patient to smile on his R side or hold air in his cheeks. You suspect your patient may have a lesion of which cranial nerve? a. CN 1 b. CN 3 c. CN 5 d. CN 7 e. CN 9

a. CN 1 olfactory - is involved with the ability to smell and has nothing to do with facial expressions. b. CN 3 oculomotor-Though this nerve is involved in eye movements, it has no control over the ability to smile or hold air in your cheeks. c. CN 5 trigeminal-Though this nerve is involved in facial sensation, it plays no part in facial musculature. d. CN 7 facial- CORRECT- supplies motor innervation to muscles of facial expression, including the orbicularis oris (smiling) and buccinator (holding air in cheeks). This nerve also supplies taste innervation to the anterior ■ of the tongue and parasympathetic innervation to the submandibular and sublingual glands. e. CN 9 glossopharyngeal- is involved in sensory, motor and autonomic innervation to the parotid glands, mediates taste to the posterior ■ of the tongue and provides sensation to the pharynx and middle ear. However, it plays no part in blinking, smiling, or holding air in your cheeks.

Mrs. Johansen is a 34-year-old woman who presents to your outpatient clinic with complaints that "the room is spinning." She reports noticing this sensation most dramatically when she is getting out of bed in the morning. Upon questioning, she reveals that she was involved in a serious car accident a few months ago. Damage to which cranial nerve may be contributing to her condition? a. CN 2 b. CN 4 c. CN 6 d. CN 8 e. CN 10

a. CN 2 optic- Though lesions involving this nerve may affect balance secondary to vision deficits, impairment in CN II is not associated with positional vertigo. b. CN 4 trochlear-Though lesions involving this nerve may affect balance secondary to vision deficits, impairment in CN IV is not associated with positional vertigo. c. CN 6 abducens-Though lesions involving this nerve may affect balance secondary to vision deficits, impairment in CN VI is not associated with positional vertigo. d. CN 8 vestibulocochlear- CORRECT-Lesions in CN VIII are associated with positional vertigo. e. CN 10 vagus- innervates muscles controlling gag reflex and autonomic muscles of heart, esophagus, trachea, stomach and small intestine. It is not associated with positional vertigo.

You are performing an initial evaluation in your outpatient clinic on a 66 year old African American female with atrial fibrillation when she suddenly begins to complain of double vision and weakness on her left side. You decide to sit the patient down and further investigate her neurologic symptoms. Other than her hemiparesis, your findings include: paresis of up/down/medial gaze, ptosis and dilated pupil of the R eye. Which cranial nerve palsy is this patient presenting with and what clinical tests would you perform in order to determine this? a. CN II: facial sensation, corneal reflex, and strength of the muscles of mastication b. CN V: facial sensation, corneal reflex, and strength of the muscles of mastication c. CN III: pupillary light reflex, accommodation reaction, and visual tracking d. CN II: pupillary light reflex, accommodation reaction, and visual fields e. CN V: pupillary light reflex, accommodation reaction, and visual tracking

a. CN II: facial sensation, corneal reflex, and strength of muscles of mastication- Deficits related to CNII may include partial or complete blindness with the visual field deficit dependent upon the area of nerve affected. The clinical tests of facial sensation and strength of the muscles of mastication would be better suited to assess the integrity of CNV. Neither CNII nor CNV are involved in the deficits of ptosis, dilated pupils and deficits in eye movements. b. CN V: facial sensation, corneal reflex, and strength of muscles of mastication- Clinical tests of Facial sensation and strength of the muscles of mastication are relevant to assess the integrity of CNV however CNV is not implicated in symptoms of ptosis, dilated pupils and deficits in eye movements. Corneal reflex testing is used to assess the integrity of CNV as well as CN VII. c. CN III: pupillary light reflex, accommodation reaction, and visual tracking- CORRECT- Oculomotor nerve palsy may present with ptosis, a dilated and fixed pupil and impairment in upward, downward and inward movements of the eye. d. CN II: pupillary light reflex, accommodation reaction, and visual field- Visual field testing would provide information regarding CN II. Cranial nerve III however, innervates the muscle constricting the pupil and the muscle that controls lens thickness thus controlling the efferent reaction of accommodation and constriction of the pupil. e. CN V: pupillary light reflex, accommodation reaction, and visual tracking- The clinical tests involved are correct for assessing the clinical presentation of the patient described above however these tests assess the integrity of CN III rather than CNV. Testing of CNV would include testing sensation of the face and strength of the muscles of mastication.

You are working with Mr. Gibbs, a 64 year old male s/p CVA, in your acute care hospital. You check the orders and notice the hospitalist ordered a full work-up following the CVA. All of the following diagnostic tests and implications are appropriately matched EXCEPT: a. Carotid ultrasound and stenosis b. Chemistry and hypoglycemia c. CBC and anemia d. ECHO and hyperglycemia e. CTA/MRA/DSA and blood supply

a. Carotid ultrasound and stenosis- these are appropriately matched b. Chemistry and hypoglycemia- these are appropriately matched c. CBC and anemia- these are appropriately matched d. ECHO and a-fib- CORRECT- ECHO is matched with a clot in the left atrium, not hyperglycemia e. CTA/MRA/DSA and blood supply- these are appropriately matched

Ms. Penny is a 58 y.o. female being treated in the NOA clinic. Her medical history states that she is also currently going through chemotherapy. She presents with hyperreflexia, spastic paralysis, loss of pain and temperature sensation on her left side and loss of proprioception on her right. What do you think is causing her symptoms? a. Cauda equina tumor from a breast cancer metastasis b. Extradural compression on the right causing Brown Sequard Syndrome from a breast cancer metastasis c. Extradural compression on the left causing Brown Sequard Syndrome from a brain cancer metastasis d. Small central lesion e. Intramedullary tumor

a. Cauda equina tumor from a breast cancer metastasis (incorrect - cauda equina tumor causes saddle anesthesia and analgesia with bladder dysfunction) b. Extradural compression on the right causing Brown Sequard Syndrome from a breast cancer metastasis (CORRECT - Brown sequard syndrome presents with hyperreflexia, spastic paralysis, contralateral loss of pain and temperature, ipsilateral loss of proprioception and is commonly caused by metastatic breast cancer) c. Extradural compression on the left causing Brown Sequard Syndrome from a brain cancer metastasis (incorrect - Brown sequard syndrome presents with hyperreflexia, spastic paralysis, contralateral loss of pain and temperature, ipsilateral loss of proprioception and is commonly caused by metastatic breast cancer) d. Small central lesion (incorrect - a small central lesion presents with loss of pain and temperature with a possible loss of proprioception and position sense) e. Intramedullary tumor (incorrect - an intramedullary tumor causes weakness and spasticity and loss of pain and temperature senses bilaterally)

Mrs. Blake, a 47 year old female, was admitted to your acute care hospital secondary to anoxic encephalopathy. You receive orders for PT evaluation and treatment. Upon examination you notice the patient is experiencing signs and symptoms consistent with rigidity. Which area of the brain may have been injured? a. Cerebellum b. Thalamus c. Hippocampus d. Basal Ganglia e. Hypothalamus

a. Cerebellum - lesion can cause hypotonia b. Thalamus - lesion can cause contralateral weakness, ataxia, and spontaneous pain c. Hippocampus - lesion can cause short term and long term memory loss d. Basal Ganglia - CORRECT- injury to this area can lead to rigidity e. Hypothalamus - bilateral lesion causes complete cessation of food intake

While working in the trauma center, your newest patient, Mr. Powderednose, has just been admitted for a hemorrhagic bleed. He is now stable and you are evaluating him for potential discharge. His chart reads that he denies any illicit drug use, but his lab values indicate there is cocaine in his system. Upon questioning him, he readily admits to cocaine use. You spoke to the attending physician prior to entering the patient's room and he indicated rampant cocaine use has probably caused his intracranial bleeding. How do you explain to your patient that cocaine may have played a major role in his current hospitalization? a. Cocaine affects the abilities of dopamine receptors, and excess dopamine in the brain may mimic a hemorrhagic stroke. b. It probably did not; cocaine does not affect blood pressure enough to affect cerebral perfusion c. Cocaine may cause vasodilation, inducing an acute hypotensive episode d. Cocaine may cause vasospasm, inducing an acute hypertensive episode e. Cocaine will not affect blood pressure, but may cause spontaneous clot formation, but only in cerebral arterioles.

a. Cocaine affects the abilities of dopamine receptors, and excess dopamine in the brain may mimic a hemorrhagic stroke. → INCORRECT. Cocaine does affect dopamine receptors, but this will probably not mimic a hemorrhagic stroke. b. It probably did not; cocaine does not affect blood pressure enough to affect cerebral perfusion. → INCORRECT. Cocaine is a known vasoconstrictor, and will usually increase blood pressure. c. Cocaine may cause vasodilation, inducing an acute hypotensive episode. → INCORRECT. Cocaine is a known vasoconstrictor, increasing blood pressure. d. Cocaine may cause vasospasm, inducing an acute hypertensive episode. → CORRECT. Cocaine, especially an 'overdose', may cause excessive vasoconstriction or vasospasm, sometimes resulting in hemorrhagic stroke. e. Cocaine will not affect blood pressure, but may cause spontaneous clot formation, but only in cerebral arterioles. → INCORRECT. Cocaine has been shown to increase clotting factors, but not specifically in the cerebral arteries. Also, cocaine does affect blood pressure.

You are in an acute care hospital treating Mrs. Tiledcieling, who has listed Arnold Chiari malformation in her past medical history. You want to know the extent to which her Chiari malformation has progressed, because you are concerned about communicating versus non-communicating hydrocephalus. What is the difference between communicating and non-communicating hydrocephalus? a. Communicating hydrocephalus allows CSF flow from the ventricles to the spinal cord, while non-communicating hydrocephalus does not allow CSF flow from the ventricles to the spinal cord. b. Non-communicating hydrocephalus does not allow CSF flow between the lateral ventricles, while communicating hydrocephalus allows CSF flow between the lateral ventricles. c. Communicating hydrocephalus has no signs or symptoms because the CSF-blood barrier in the choroid plexus is intact, while in non-communicating hydrocephalus the blood-CSF barrier is damaged, so patients may present with generalized hypoxemia to the neural tissues, resulting in an array of neurological signs and symptoms. d. Non-communicating hydrocephalus has no signs or symptoms because the CSF-blood barrier in the choroid plexus is intact, while in communicating hydrocephalus the blood-CSF barrier is damaged, so patients may present with generalized hypoxemia to the neural tissues, resulting in an array of neurological signs and symptoms. e. There is no difference between the two; patients may present with neurological symptoms regardless of communicating vs. non-communicating hydrocephalus.

a. Communicating hydrocephalus allows CSF flow from the ventricles to the spinal cord, while non-communicating hydrocephalus does not allow CSF flow from the ventricles to the spinal cord. CORRECT - non-communicating hydrocephalus refers to the inability of the CSF to reach the spinal cord due to obstruction of the lower ventricular system. b. Non-communicating hydrocephalus does not allow CSF flow between the lateral ventricles, while communicating hydrocephalus allows CSF flow between the lateral ventricles. INCORRECT - non communicating hydrocephalus is primarily involved with the ability of the CSF to reach the spinal cord, due to a blockage in the lower ventricular system. c. Communicating hydrocephalus has no signs of symptoms because the CSF-blood barrier in the choroid plexus is intact, while in non-communicating hydrocephalus, the blood-CSF barrier is damaged, so patients may present with generalized hypoxemia to the neural tissues, resulting in an array of neurological signs and symptoms. INCORRECT Hydrocephalus does not affect the CSF-blood barrier in the choroid plexus. d. Non-communicating hydrocephalus has no signs or symptoms because the CSF-blood barrier in the choroid plexus is intact, while in communicating hydrocephalus, the blood-CSF barrier is damaged, so patients may present with generalized hypoxemia to the neural tissues, resulting in an array of neurological signs and symptoms. INCORRECT. Hydrocephalus does not affect the CSF-blood barrier in the choroid plexus. e. There is no difference between the two; patients may present with neurological symptoms regardless of communicating vs. non-communicating hydrocephalus. INCORRECT. There is a difference, that being the ability of the CSF to reach the entire subarachnoid space, including the spinal cord(communicating), vs. the inability of the CSF to reach the entire subarachnoid space, including the spinal cord(non-communicating).

It is the first day of your pediatrics affiliation at CHOP and you are working in the newborn baby gym. You first patient is a little girl who has been diagnosed with meningomyelocele. You see that the doctors have ordered further testing, but there is no explanation on the script for why further examination is being performed. However, you remember that meningomyelocele is often seen in children with ________ and can be associated with _________. a. Congenital hips, chronic hip dislocations b. Chiari type II malformation, hydrocephalus c. Spina bifida occulta, abnormal vertebral arches d. Chiari type I malformation, hydrocephalus e. Gi issues, cholecystitis

a. Congenital hips, chronic hip dislocations - although congenital hips may be associated with chronic hip dislocations and are typically found in newborns, they are not associated with meningomyelocele. b. Chiari type II malformation, hydrocephalus - CORRECT meningomyelocele is often associated with Chiari type II malformations and can lead to hydrocephalus due to the caudal displacement of the medulla, cervicomedullary junction, pons, 4th ventricle and low cerebellar tonsils. c. Spina bifida occulta, abnormal vertebral arches - spina bifida occulta is a result of a defect in one or more of the vertebral arches, but the spinal cord and meninges are normal. In meningomyelocele, the spinal cord and meninges are herniated through a vertebral defect. d. Chiari type I malformation, hydrocephalus - although hydrocephalus can be associated with Chiari type I and type II malformations, Chiari type II malformations are associated with meningomyelocele (not Chiari type I). e. GI issues, cholecystitis - although cholecystitis is a GI issue, it is not associated with meningomyelocele

Mr. Atwood is a 22-year-old male who sustained a complete C7 SCI following a severe MVA 8 months ago. You work at UDel Rehab and are to evaluate him this morning. Before doing so, you review his past medical history and see that (among other things) he has had several gastrointestinal issues. Based on your superior knowledge of SCI management issues, you might expect to see all of the following listed except: a. Constipation b. GI bleeding c. Cholecystitis d. Chronic diarrhea e. Pancreatitis requiring surgical intervention

a. Constipation - due to decreased motility; may be treated with stool softeners, motility agents, suppositories. b. GI bleeding - can occur due to increased gastric acid secretion and decreased sympathetically-mediated vasoconstriction. c. Cholecystitis - due to abnormal gallbladder motility and biliary secretions, impaired enterohepatic circulation, as well as increased risk of gallstones. d. Chronic diarrhea - CORRECT; although fecal incontinence can occur following SCI, frequent, watery stools are not common unless there is another underlying issue. e. Pancreatitis requiring surgical intervention - due to elevated blood calcium levels and increased viscosity of pancreatic secretions; more common in the first month after SCI.

one of your patients being treated for a brain tumor. You noticed these two images. Having a thorough grasp on your neurologic imaging studies, you know that image A is a/an __________ used to measure __________. You are relieved to see that tracks in image B, a ________ study, are being displaced, meaning that ________. a. DTI; blood flow to portions of the brain during certain tasks; fMRI; the tracks are involved in the tumor b. fMRI; tumor presence in different portions of the brain; DTI; the tumor is quickly growing c. DTI; blood flow to the tumor; fMRI; the tumor is benign d. DTI; blood flow towards the recording device; fMRI; the tracks surround the tumor are hyperactive e. fMRI; blood flow to portions of the brain during certain tasks; DTI; the tracks are not

a. DTI; blood flow to portions of the brain during certain tasks; fMRI; the tracks are involved in the tumor - Incorrect test; correct statement; incorrect test; if the tracks are being displaced, it means that they are not involved in the tumor b. fMRI; tumor presence in different portions of the brain; DTI; the tumor is quickly growing - Correct test; fMRI's would not be the test used to detect tumor presence; Correct test; DTI's do no track speed of tumor growth c. DTI; blood flow to the tumor; fMRI; the tumor is benign - Incorrect test; this test does not measure blood flow specifically to a tumor; Incorrect test; this test would not be used to determine malignancy of a tumor d. DTI; blood flow towards the recording device; fMRI; the tracks surround the tumor are hyperactive - Incorrect test; this test shows blood flow to an area of the brain, not to and from a recording device (Duplex Scans would show this); Incorrect test; tracks being displaced does not mean they are hyperactive e. fMRI; blood flow to portions of the brain during certain tasks; DTI; the tracks are not involved in the tumor - CORRECT The first image is an fMRI and the second image is a DTI. fMRI's measure blood flow to portions of the brain during certain tasks. If the tracks in a DTI scan are being displaced, it means that the tracks are not involved in the tumor.

While working in an outpatient physical therapy clinic you are treating a 23 year old male, Kyle, for low back pain. This is your first treatment after the evaluation and you notice a few changes in the patient's mood. He demonstrates a tense posture, restlessness, and is fidgeting his fingers while waiting for you. Which mood disorder do you suspect? a. Depression b. Anxiety c. Mania d. Bipolar disorder e. Dysthymia

a. Depression - associated with depressed mood, changes in eating and sleeping patterns, loss of energy and initiative, low self-esteem, poor concentration, lack of enjoyment of previously pleasurable activities, self-destructive or suicidal thoughts and behavior b. Anxiety - CORRECT - associated with tense posture, restlessness, and fidgeting c. Mania - abnormally active and disorganized d. Bipolar disorder - described by alternating periods of mania and depression e. Dysthymia - reports a low mood almost daily over a span of at least 2 years

You are treating a 60-year-old male patient who sustained a SCI about 1 month ago in a car accident. He noted that he had also injured his shoulder and left hip in the accident. Now, he has increased complaints of pain at his hip, decreased ROM, increased spasticity and the area is firm upon palpation. You have ruled out a DVT and are considering heterotopic ossification as his diagnosis. What is the best early diagnostic test and following treatment that would be indicated if heterotopic ossification is suspect? a. Diagnosis = Triple-phase bone scan; Treatment = bisphosphonate and possibly surgery after 12-18 months b. Diagnosis = X-ray; Treatment = immediate surgery c. Diagnosis = Alkaline phosphatase; Treatment = total joint arthroplasty d. Diagnosis = MRI; Treatment = calcium channel blockers e. Diagnosis = There is no good test when the patient is this acute; Treatment = Maintain remaining ROM because there is not current treatment that will effectively regain lost ROM

a. Diagnosis = Triple-phase bone scan; Treatment = bisphosphonate and possibly surgery after 12-18 months - CORRECT - A bone scan may be the only definitive test and will show heterotopic ossification 7-10 days earlier than x-ray. The triple-phase scan is likely the most sensitive method; some form of bisphosphonate is a common treatment for heterotopic ossification as it can act as a calcium antiresorptive agent. Surgery may or may not be indicated and could be considered after 12-18 months. b. Diagnosis = X-ray; Treatment = immediate surgery - an x-ray may not show heterotopic ossification in its early phases because there may not be enough calcium in the matrix (x-ray will show heterotopic ossification later but is not the best early diagnostic test; surgery may be needed, but the best treatment would be etidronate followed by surgery after 12-18 months. c. Diagnosis = Alkaline phosphatase; Treatment = total joint arthroplasty - Alkaline phosphatase will be elevated eventually, but may not be elevated initially; this is a drastic treatment - the joint may not need to be replaced and could be treated conservatively with bisphosphonate, possibly without surgery. d. Diagnosis = MRI; Treatment = calcium channel blockers - an MRI is primarily used to diagnose soft tissue injuries and may not be the best test to differentiate heterotopic ossification; calcium channel blockers are used for dysrhythmias, not heterotopic ossification e. Diagnosis = There is no good test when the patient is this acute; Treatment = Maintain remaining ROM because there is not current treatment that will effectively regain lost ROM - A triple-phase bone scan will show heterotopic; there are effective treatments for heterotopic ossification.

You are an acute care PT getting ready to perform an assessment on a 41 year old patient who was just diagnosed with acquired Chiari type I malformation. You know that, when you go into his room, you will need to avoid standing off to his side and give him clear (and possibly loud) verbal instructions, rather than simply demonstrating a task. You do this because you know that common symptoms of Chiari type I include all of the following EXCEPT: a. Diplopia or blurred vision b. Tinnitus c. Lower cranial nerve dysfunction d. Nystagmus e. Cataracts

a. Diplopia or blurred vision - This is a common symptoms of Chiari type I, therefore, patients may need more auditory cues and instructions to understand a task b. Tinnitus - This is a common symptoms of Chiari type I, therefore, you may need to speak loudly and clearly when giving instructions to a patient c. Lower cranial nerve dysfunction - This is a common symptoms of Chiari type I, therefore, patients may have visual impairments and need more auditory cues d. Nystagmus - This is a common symptoms of Chiari type I, therefore, patients may need more auditory cues and instructions to understand a task e. Cataracts - CORRECT - While this is a good reason to perform an assessment in this fashion, cataracts are not a common symptom of Chiari type I malformation

You have orders to eval and treat Mrs. Feldman, a 72 year old female s/p right CVA, in the acute rehab hospital. You notice she does not look at you when you stand to her left, and she is sitting on her left arm, leading you to believe she has neglect. All of the following are treatments for neglect EXCEPT: a. Eye patching b. Visual scanning c. Visual and verbal cues to remind patient to attend to affected side d. Virtual reality environments e. Surgical interventions

a. Eye patching- this is a treatment for neglect b. Visual scanning- this is a treatment for neglect c. Visual and verbal cues to remind patient to attend to affected side- this is a treatment for neglect d. Virtual reality environments- this is a treatment for neglect e. Surgical interventions- CORRECT- this is not a treatment for neglect

Josh Seiner is a 90 y.o male who is expected to arrive at your outpatient clinic today. During a chart review you notice MPH of an intraparenchymal stroke and hypertension. As an educated physical therapy student you suspect that the underlying cause of this stroke (besides hypertension) was _________________. a. Edema b. Prominent ventricles c. Heart attack d. Whip-lash e. amyloid angiopathy

a. Edema- although edema can lead to ischemia and therefore stroke, given the history, it is unlikely that this was the cause b. Prominent ventricles- though this is a sign of aging, it would not lead to intraparenchymal stroke c. Heart attack - hypertension is a risk factor for heart attack, but this answer does not make sense for the question being asked d. Whip-lash- can lead to a TBI and possibly edema, but there is no history of a traumatic event to lead to this conclusion e. amyloid angiopathy- CORRECT- given the pt's age and history of high blood pressure it is likely the hardened vessels "cracked" due to pressure

You are an acute care PT working at Children's Hospital of Philadelphia, when you see a mother bring in her 4 month old child. You hear her say that her baby has been much more irritable than usual, and she has not been able to calm him down. He also has a fever. As a well educated PT, you suspect that the child has _______ and a _______ should be performed: a. Encephalopathy; Lumbar puncture at L4-L5 level b. Subarachnoid hemorrhage; Lumbar puncture at L4-L5 level c. Subarachnoid hemorrhage; MRI d. Encephalopathy; CT scan e. Encephalopathy; MRI

a. Encephalopathy; Lumbar puncture at L4-L5 level - CORRECT High irritability is a characteristic of encephalopathy in an infant and this can be diagnosed by a lumbar puncture b. Subarachnoid hemorrhage; Lumbar puncture at L4-L5 level - Although a lumbar puncture is the correct test, you are not suspecting subarachnoid hemorrhage as a result of hyperirritability. c. Subarachnoid hemorrhage; MRI - You should not be suspecting subarachnoid hemorrhage from hyperirritability, and an MRI would not tell you the correct information. d. Encephalopathy; CT scan - Hyperirritability is a sign of encephalopathy, but a CT scan is not the correct test to be performed because it will not allow you to assess CSF. e. Encephalopathy; MRI - Hyperirritability is a sign on encephalopathy, but an MRI is not the correct test to be performed because it will not allow you to assess CSF.

Mr. Keffer is a 60 year old teacher presenting to outpatient PT for recent deterioration of balance. This is his sixth PT session. You are re-evaluating him when he notices numbness on the left side of his face and entire right forearm and hand. You also notice signs of ataxia and nystagmus, but his mental status appears perfectly normal. You decide to refer him for further evaluation. Which of the following imaging techniques would most likely detect physiological deficits associated with Mr. Keffer's symptoms: a. FLAIR MRI b. CT in brain window c. Arteriography d. Diffusion weighted MRI e. Ultrasonography

a. FLAIR MRI - This window for MRI is good at detecting edema of the brain. b. CT in Brain window - This imaging is not good at detecting hemorrhages under 1 cm. - this is not necessarily true. c. Arteriography - this is used primarily for detecting medium-large stenotic/occluded arteries in the head and neck d. Diffusion weighted MRI - CORRECT - good at detecting stroke in small areas (brainstem, cerebellum) e. Ultrasonography - this is primarily for detecting stenosis of arteries in the neck and some intracranial vessels

You are working in an acute care hospital, and your patient, 30 year old Mrs. Extensioncord, has listed in her chart that she is affected by the most common hereditary hypercoagulopathy. This coagulopathy is known as _______. a. Factor V b. Factor V Leiden c. Koagulations-Vitamin deficiency d. Factor IX(or Christmas factor) e. Von Willebrand factor

a. Factor V INCORRECT - this disorder refers to hypocoagulability, not hypercoagulability b. Factor V Leiden CORRECT - this is the most common hereditary hypercoagulability c. Koagulations-Vitamin deficiency INCORRECT - this is the German and Scandinavian root of vitamin K, referring to its ability to increase coagulability. A deficiency of vitamin K would actually decrease coagulability, not increase d. Factor IX(or Christmas factor) INCORRECT - This is correlated with hypocoagulability, not hypercoagulability e. Von Willebrand factor INCORRECT- this disorder will also decrease coagulability, not increase

Mr. Jones is a 67 year old African American male you are currently treating in an outpatient clinic for insidious onset low back pain likely secondary to his obesity and low activity level. After working on gait training for 15 minutes today, he begins to complain of weakness in his left arm and leg. Taking advantage of your high quality University of Delaware education, you're thinking... a. Fatigue is normal with a sedentary adult and he doesn't present with any of the risk factors for stroke. b. The magic window for IV rt-PA is 6-12 hours after onset, so I should finish my treatment and then send him to the emergency room. c. He's definitely having a stroke, I'll add two units of neuro re-ed to today's treatment! d. IV rt-PA needs to be administered as soon as possible, but after a CT scan, of course. e. His symptoms are consistent with neurogenic claudication and should resolve by itself when he sits down.

a. Fatigue is normal with a sedentary adult and he doesn't present with any of the risk factors for stroke. - Upper extremity fatigue is not likely to be explained by the gait training and stroke risk factors of male, African American, obesity, and sedentary lifestyle all put him at risk for a stroke b. The magic window for IV rt-PA is 6-12 hours after onset, so I should finish my treatment and then send him to the emergency room. - IV rt-PA needs to be administered < 4.5 hours after onset (preferably < 3) to limit the risks of hemorrhagic conversion c. He's definitely having a stroke, I'll add two units of neuro re-ed to today's treatment! - The therapist is able to correctly identify the symptoms, but incorrectly continues physical therapy treatment when immediate medical attention is required d. IV rt-PA needs to be administered as soon as possible, but after a CT scan, of course. - CORRECT - Clot busting IV rt-PA medications should be administered as soon as possible, but not > 4.5 hours after symptom onset, once a CT scan is performed to rule out a hemorrhagic stroke e. His symptoms are consistent with neurogenic claudication and should resolve by itself when he sits down - Lower extremity weakness is not impossible with neurogenic claudication, but the upper extremity weakness would not be explained

You have just started your job as a PT in the acute care setting at UDel Hospital. Your first patient on your first day is a 76-year-old gentleman named Jed Kleiss, who was admitted yesterday s/p ischemic stroke. During your evaluation/examination you would like to see if a pronator drift is present. What would this tell you about Mr. Kleiss' condition if this test was positive? Which side would the drift be present on? a. He may have a lesion which ultimately affects his corticospinal tract. It would be present on the contralateral side of the lesion. b. He may have a lesion which ultimately affects his spinothalamic tract. It would be present on the contralateral side of the lesion. c. He may have a lesion which ultimately affects his corticospinal tract. It would be present on the ipsilateral side of the lesion. d. He may have a lesion which ultimately affects his spinothalamic tract. It would be present on the ipsilateral side of the lesion. e. He may have a lesion which ultimately affects his coordinopronator tract. It would be present on the ipsilateral side of the lesion.

a. He may have a lesion which ultimately affects his corticospinal tract. It would be present on the contralateral side of the lesion. - CORRECT. The corticospinal tract allows volitional movement and decussates in the medulla oblongata. Therefore, the right cerebral cortex controls movement on the left side of the body, and vice versa. Pronator drift is indicative of an UMN lesion. b. He may have a lesion which ultimately affects his spinothalamic tract. It would be present on the contralateral side of the lesion. - The spinothalamic tract carries pain and temperature sensory information and is not the correct choice, but the pronator drift would be present on the contralateral side of the lesion. c. He may have a lesion which ultimately affects his corticospinal tract. It would be present on the ipsilateral side of the lesion. - The corticospinal tract is correct, but the pronator drift would not be present on the ipsilateral side of the lesion. d. He may have a lesion which ultimately affects his spinothalamic tract. It would be present on the ipsilateral side of the lesion. - This is neither the correct tract nor the correct side. e. He may have a lesion which ultimately affects his coordinopronator tract. It would be present on the ipsilateral side of the lesion. - The coordinopronator tract is not a real tract, and the pronator drift would not be present on the ipsilateral side of the lesion.

You are working in the UDPT NOA clinic with Mr. Fry, a 35 year old male, who recently suffered a TBI from a motorcycle accident. You bring your patient back from the waiting room and notice an unusual gait pattern that looks like a pair of scissors. Which of the following is NOT a characteristic of scissoring gait pattern? a. Increased tone in the hip adductors b. Short step length c. Stiff gait pattern d. Thigh crossover e. Decreased tone in the hip adductors

a. Increased tone in the hip adductors - this is a characteristic of scissoring gait b. Short step length - this is a characteristic of scissoring gait c. Stiff gait pattern - this is a characteristic of scissoring gait d. Thigh crossover - this is a characteristic of scissoring gait e. Decreased tone in the hip adductors - CORRECT - scissor gait has an increase in tone

Ms. Jones is a 85 year old caucasian female you are working with on pre-prosthetic training in outpatient rehab after her recent DM induced peripheral neuropathy lead to a Syme's amputation. During treatment today, you notice she is slurring her speech and the right side of her face is drooping. You correctly realize these are signs Ms. Jones is having a stroke, but remember there are certain exclusions for rt-PA and in quickly scanning her chart, in addition to being > 85 years old and having DM, rt-PA is not appropriate because... a. Her last blood pressure reading was 110/70 b. She has a previous history of intracranial hemorrhage c. Heparin was administered 96 hours ago d. She had a myocardial infarction 4 years ago e. She had her knee replaced 8 years ago

a. Her last blood pressure reading was 110/70 - SBP > 185 mm or DBP > 110 mm exclude the use of rt-PA. b. She has a previous history of intracranial hemorrhage - CORRECT - intracranial hemorrhage is an exclusion to administering rt-PA c. Heparin was administered 96 hours ago - Heparin is an exclusion when administered < 48 hours previously d. She had a myocardial infarction 4 years ago - MI is an exclusion if it occurred within the last 3 months e. She had her knee replaced 8 years ago - Major surgery is only a contraindication if < 14 days ago

While treating in the hospital your 82 year old African American patient Mr. Jones cannot stop talking about his cooking and how much he loves to eat fried foods. From his chart, you notice he has a significant history of cardiovascular complications including significant bilateral stenosis (due to atherosclerotic plaque) of his internal carotid arteries and congestive heart failure. He also has a long history of radiation therapy to his neck. Your initial evaluation is interrupted by the nurse, who informs you that your patient will be undergoing a right CEA later this afternoon, effectively ending your session with him. You know, however, that he is at high risk for having this procedure performed. Which of the following factors does NOT put Mr. Jones at increased risk both during and after a right carotid endarterectomy? a. His age b. His left internal carotid occlusion c. The fact that he is an African American d. His previous history of radiation therapy e. Congestive heart failure

a. His age INCORRECT. Age >80 does put him at increased risk both during and after a CEA b. His left internal carotid occlusion INCORRECT. His contralateral ICA occlusion puts him at increased risk both during and after a CEA. c. The fact that he is an African American CORRECT. Race has not been correlated with increased risk both during and after a CEA. d. His previous history of radiation therapy INCORRECT. Prolonged radiation therapy to his neck does put him at increased risk both during and after a CEA. e. Congestive heart failure INCORRECT. CHF is a risk factor for patients undergoing CEA.

You are treating a 17-year-old male in the hospital setting for an acute SCI. When you have treated this patient previously, he had been easy to work with, had a calm demeanor, stable vitals, and was generally not a "complainer". When you walk in the room today, you see that he does not look like his normal self and he informs you that he, "doesn't feel well and is really nervous". You notice several symptoms that cause you to believe that your patient may be experiencing Autonomic Dysreflexia. Which of the following would NOT lead you to think the patient is experiencing AD? a. His lesion is at T5 b. His BP has increased from 120/80 mmHg to 180/90 mmHg c. His HR has increased from 60 bpm to 110 bpm d. He is sweating profusely and looks "clammy" e. You know that he is catheterized

a. His lesion is at T5 - Lesions at or above T6 increase risk for AD b. His BP has increased from 120/80 mmHg to 180/90 mmHg - HTN > 20-40 mmHg above baseline or > 150 mmHg is a sign of AD c. His HR has increased from 60 bpm to 110 bpm - CORRECT - in AD, the parasympathetic system tries to compensate from the spike in BP (caused by the sympathetic nervous system), by actually lowering HR. d. He is sweating profusely and looks "clammy" - this is a sign of AD e. You know that he is catheterized - AD can come from a noxious stimuli below the level of the lesion, such as a kink in the catheter

Mr. Kent is a 70 year old man who presents to the NOA clinic with a left sided stroke. MRI shows the lateral area of his primary motor cortex was affected by the infarct. While working on the treadmill (his least favorite activity), he sticks his tongue out at you. You notice that his tongue does what? a. His tongue is atrophied and deviates to the left. b. His tongue deviates to the right with no atrophy. c. His tongue is atrophied and deviates to the right. d. His tongue deviates to the left with no atrophy. e. His tongue curls.

a. His tongue is atrophied on the left and deviates to the left - LMN deviates toward side of atrophy b. His tongue deviates to the right with no atrophy- CORRECT- UMN (stroke) will have no atrophy but deviation contralateral to UMN lesion c. His tongue is atrophied on the left and deviates to the right - LMN deviates toward side of atrophy d. His tongue deviates to the left with no atrophy- UMN (stroke) will have no atrophy but deviation contralateral to UMN lesion e. His tongue curls - UMN (stroke) will have no atrophy but deviation contralateral to UMN lesion

You are evaluating a patient who had a spinal cord injury at the T6 level. Upon examination, you observe some deep tendon reflex changes and mild atrophy of the trunk musculature. You want to examine the patient's abdominal strength, so you ask him to perform a sit-up. You notice that as the patient flexes his neck, his umbilicus is pulled upward. As a UDPT student, you know that this is called _______ and occurs because _________. a. Hoffman's sign: lower portion of rectus abdominus is intact and upper portion of rectus abdominis is weak due to SCI at T6 level. b. McMurray's sign: this is normal. c. Beevor's sign: upper portion of rectus abdominis is intact and lower portion of rectus abdominis is weak due to SCI at T6 level. d. Babinski's sign: upper portion of rectus abdominis is intact and lower portion of rectus abdominis is weak due to SCI at T6 level. e. O'Briens sign: this is normal.

a. Hoffman's sign: lower portion of rectus abdominus is intact and upper portion of rectus abdominis is weak due to SCI at T6 level. - Hoffman's sign is a test for UMN lesion and does not have anything to do with the rectus abdominus and umbilicus. b. McMurray's sign: this is normal. - McMurray's is a special test for the knee. c. Beevor's sign: upper portion of rectus abdominis is intact and lower portion of rectus abdominis is weak due to SCI at T6 level. - CORRECT Beevor's sign occurs when the upper portion of the rectus abdominus is intact and the lower portion is weak due to SCI at the T6 spinal level, which causes the umbilicus to pull upward during a sit up. d. Babinski's sign: upper portion of rectus abdominis is intact and lower portion of rectus abdominis is weak due to SCI at T6 level. - The reasoning is correct, but Babinski is a test for UMN lesions and does not have anything to do with the umbilicus. e. O'Briens sign: this is normal. - O'Brien's is a special test for the shoulder.

You are at your elective affiliation at A.I. duPont. You are to perform a PT consult for a 4-month-old boy with cerebral palsy and need to check some of the common pediatric reflexes during your evaluation. Among other things, you would like to check his Moro reflex. How would you go about doing this and what response would you expect in a typical infant of this age? a. Hold the child in a prone position and simulate a sudden loss of support; extension of arms and legs followed by a drawing back of arms to midline. b. Hold the child in a supine position and simulate a sudden loss of support; extension of arms and legs followed by a drawing back of arms to midline. c. Hold the child in a prone position and simulate a sudden loss of support; flexion of arms and legs followed by a return to resting position. d. Hold the child in a supine position and simulate a sudden loss of support; flexion of arms and legs followed by a return to resting position. e. Hold the child in a sitting position and abduct their arms overhead; extension of arms and legs followed by a drawing back of arms to midline.

a. Hold the child in a prone position and simulate a sudden loss of support; extension of arms and legs followed by a drawing back of arms to midline. - This is an incorrect position/procedure but the correct typical response (extension of arms and legs followed by a drawing back of arms to midline). b. Hold the child in a supine position and simulate a sudden loss of support; extension of arms and legs followed by a drawing back of arms to midline. - CORRECT. The Moro reflex is "a normal reflex for an infant when he or she is startled or feels like they are falling. The infant will have a 'startled' look and the arms will fling out sideways with the palms up and the thumbs flexed. As the reflex ends, the infant draws its arms back to the body, elbows flexed, and then relaxes" (from the A.D.A.M. medical encyclopedia). c. Hold the child in a prone position and simulate a sudden loss of support; flexion of arms and legs followed by a return to resting position. - This is an incorrect position/procedure and incorrect typical response. d. Hold the child in a supine position and simulate a sudden loss of support; flexion of arms and legs followed by a return to resting position. - This is the correct position/procedure (supine and simulating a sudden loss of support) but an incorrect typical response. e. Hold the child in a sitting position and abduct their arms overhead; extension of arms and legs followed by a drawing back of arms to midline. - This is an incorrect position/procedure but the correct typical response.

You are taking over the subacute care of a TBI patient because his normal therapist is going on paternity leave. He was injured in a motorcycle accident, the motorcycle was estimated to be going 65 mph, and he was also drunk. After the injury he was unconscious for 10 hours, it was estimated that he lost 2 pints of blood during emergency surgery, he was able to recall daily events 5 days after the accident, he was not able to verbally respond to questions for 3 days, he had an abnormal CT and he was able to open his eyes right away and he had a GCS score of 11. His family is asking you how bad it looks. What are the 3 factors you are looking at in determining your answer? a. How fast the vehicle was going on impact, how much alcohol was consumed, the length of time the patient was unconscious. b. The amount of time post traumatic amnesia is experienced, if the patient can communicate or not, how much blood was lost. c. How much blood was lost, GCS score, how quickly the patient can recall daily events d. The type of vehicle being driven in the accident, CT results, GCS score e. The amount of time post traumatic amnesia is experienced, GCS score, the length of time the patient was unconscious.

a. How fast the vehicle was going on impact, how much alcohol was consumed, the length of time the patient was unconscious - Length of time patient lost consciousness is one of the factors but the other two are not. b. The amount of time post traumatic amnesia is experienced, if the patient can communicate or not, how much blood is lost is not one of the factors. Communication level is part of the GCS but it is the whole GCS score that you are looking for. Time of PTA is one of the factors. c. How much blood was lost, GCS score, how quickly amnesia is experienced - GCS score and how quickly the patient can recall daily events (PTA) are two of the factors but blood loss is not. d. The type of vehicle being driven in the accident, CT results, GCS- CT results assists in severity grading as does the GCS score, however the type of vehicle does not. e. The amount of time post traumatic amnesia is experienced, GCS, the length of time patient was unconscious- CORRECT - These three are the main factors to review when assessing severity. Amount of PTA correlates to cognitive recovery, length of LOC is associated with permanent disability and GCS score is correlated with outcome, need for neurosurgical intervention and mortality.

You are working in a hospital and your patient Howard Wolowitz, a 73 year old male, was admitted for treatment of his congestive heart failure. During case rounds, a doctor mentions that Howard's mother, Mrs. Wolowitz, recently died from congestive heart failure. A radiologist notes that they observed apical ballooning of his left ventricle which may be caused by excessive sympathetic discharge likely caused by grief from his mother's recent passing, resulting in CHF. The radiologist also notes that Mr. Wolowitz had recently undergone a full cardiac stress test stress test 6 months ago, and showed no cardiac pathology. What condition is the radiologist referring to? a. Hypertrophic cardiomyopathy b. Reflex sympathetic dystrophy syndrome c. Takotsubo's (Broken Heart syndrome) cardiomyopathy d. Ventricular fibrillation e. Congenital congestive heart failure

a. Hypertrophic cardiomyopathy → INCORRECT. Wolowitz's recent cardiac workup would have shown some evidence of HCM, as this is a progressive disease and generally takes more than 6 months to manifest symptoms. b. Reflex sympathetic dystrophy syndrome → INCORRECT. This would not likely have a symptomatic effect on the cardiac tissue. c. Takotsubo's cardiomyopathy → CORRECT. Broken heart syndrome, or Takotsubo's cardiomyopathy is caused by excessive sympathetic discharge and is characterized by left ventricular apical ballooning. d. Ventricular fibrillation → This may be associated with progressive CHF, however, his previous workup combined with his apical ballooning would eliminate this from consideration, or at least make it a differential zebra. e. Congenital congestive heart failure → INCORRECT. While his mother was affected by CHF, his recent cardiac stress test without pathology decreases the likelihood of this, making it a differential zebra.

Mr. P. Athology is a 26 year old man whom you are about to treat in your acute care rotation. The patient sustained a knife wound through the back that completely transected the right half of his spinal cord; the left side of the spinal cord remains completely intact. Based on this mechanism of injury, before entering the room, you can predict that he will present with the following signs and symptoms: I. Right sided (ipsilateral) spastic paralysis below the level of the lesion II. Left sided (contralateral) loss of pain and temperature sensation III. Right sided (ipsilateral) loss if fine touch and vibration sensation IV. Right sided (ipsilateral) loss of pain and temperature a. I, II, and III b. I, III, and IV c. II, III, and IV d. Pt would present with none of these symptoms. e. I, II, III, and IV

a. I, II, and III ; CORRECT! Corticospinal tracts supplying motor cross over above the level of the lesion at the medulla. Spinothalamic tracts in the ALS system cross over AT the same spinal level, most below the level of the lesion. The DCML pathways carrying touch and vibration sensation cross over above the level of the lesion b. I, III, and IV; incorrect. Ipsilateral pain and temperature will still be intact because the tracts cross below the level of the lesion c. II, III, and IV; incorrect; IV is incorrect for the same reasons stated above. d. Pt would present with none of these symptoms. -- I, II, and III are true. e. I, II, III, and IV -- IV is not true (see above explanation).

Although every patient is unique and may vary in his/her abilities despite having a similar injury, what functional outcome could you reasonably expect for the above patient (Mr. Atwood, C7 complete SCI)? a. Independent except for a car with hand controls or adapted van b. Lower extremity dressing may be independent with equipment; car driving with hand controls or adapted van; otherwise independent c. Assistance with lower extremity dressing, bladder; van-adapted driving; possibly independent with transfer board; otherwise independent d. Assistance required with upper extremity dressing, bed mobility, weight shifts, and transfers; independent feeding and power chair use; short distances in manual chair on level surfaces with lugs or plastic rims e. May be able to feed with adaptation, as well as independence with a power chair

a. Independent except for a car with hand controls or adapted van - this is likely more typical of a C8-T1 injury. b. Lower extremity dressing may be independent with equipment; car driving with hand controls or adapted van; otherwise independent - CORRECT; this is a reasonable expectation for a C7 injury. c. Assistance with lower extremity dressing, bladder; van-adapted driving; possibly independent with transfer board; otherwise independent - this is a reasonable expectation for a C6 injury. d. Assistance required with upper extremity dressing, bed mobility, weight shifts, and transfers; independent feeding and power chair use; short distances in manual chair on level surfaces with lugs or plastic rims - this is a reasonable expectation for a C5 injury. e. May be able to feed with adaptation, as well as independence with a power chair - this is a reasonable expectation for a C3-C4 injury.

You are working in an acute care hospital with a 16 year old female swimmer, Gabriella, who suffered a SCI following a diving accident. While assessing Gabriella's lower extremity ROM, you notice a significant limitation in her available range. You suspect spasticity may be contributing to her ROM limitation. Which of the following is NOT true about spasticity? a. It is the most common abnormality seen in patients with stoke, TBI, and SCI b. SCI patients commonly demonstrate spasticity of their flexor muscles in both the upper and lower extremities c. Spasticity is not painful or accompanied by muscle spasms d. TBI patients commonly demonstrate spasticity of the flexor muscles of the upper extremity and extensor muscles of the lower extremity e. Spasticity secondary to the corticospinal tract injury is often accompanied by muscle weakness, hyper-reflexia, and an extensor plantar reflex response

a. It is the most common abnormality seen in patients with stoke, TBI, and SCI - this is true of spasticity b. SCI patients commonly demonstrate spasticity of their flexor muscles in both the upper and lower extremities - this is true of spasticity c. Spasticity is not painful or accompanied by muscle spasms - CORRECT - spasticity IS associated with pain and muscle spasms d. TBI patients commonly demonstrate spasticity of the flexor muscles of the upper extremity and extensor muscles of the lower extremity- this is true of spasticity e. Spasticity secondary to the corticospinal tract injury is often accompanied by muscle weakness, hyper-reflexia, and an extensor plantar reflex response - this is true of spasticity

Mr. H. Emorage is a 44 year old male whom you are to evaluate during your acute care internship. On his imaging report you note that he is documented as having a slowly progressive centrally located syringomyelia in his spinal cord. Given this diagnosis, which of the following would you least expect to see upon examination with this lesion? a. LMN weakness in involved segments b. UMN dysfunction below lesion c. Increased DTRs below the level of the lesion d. loss of pain and temperature sense below level of lesion bilaterally e. loss of position and vibration sense

a. LMN weakness in involved segments- this may be a characteristic of a central spinal cord lesion b. UMN dysfunction below lesion- this may be a characteristic of a central spinal cord lesion c. Increased DTRs below the level of the lesion -- this is an early sign of central cord lesions that occurrs with damage to the descending corticospinal tract. d. loss of pain and temperature sense below level of lesion- this may be a characteristic of a central spinal cord lesion e. loss of position and vibration sense- CORRECT. The DCML pathways are affected last when the lesion is in its most progressed stage.

Ms. Maykodah is your next patient you are seeing in the hospital. You open her chart beforehand and under PMHx, you see she suffered a lacunar stroke. You know some facts about lacunar strokes that you are reviewing before you go into her room. Which of the following is a FALSE fact regarding lacunar strokes? a. Lacunars are rarely greater than 10mm in diameter. b. Patients with symptoms of a lacunar stroke who are not diagnosed with imaging are said to have "lacunar stroke syndrome." c. They are associated with hypertension. d. They occur in the deep perforating arteries of the brain. e. The most common corresponding lesion occurs in the pons.

a. Lacunars are rarely greater than 10mm in diameter. - This is true. b. Pts with symptoms of a lacunar stroke who are not diagnosed with imaging are said to have "lacunar stroke syndrome."- This is true. c. They are associated with chronic HTN. - This is true. d. They occur in the deep perforating arteries of the brain. - This is true. e. The most common corresponding lesion occurs in the pons - CORRECT- This is false. The most common lesions occur in the putamen, followed by the thalamus, caudate, pons, and internal capsule.

You are in an acute care setting and one of the patients on your floor seems to be having repeated seizures, yet the MDs and nurses pay little attention. The patient is a 17-year-old female with a history of attention-seeking behavior. After witnessing this patient during one of her episodes you quickly realize it is likely a pseudoseizure due to a conversion disorder. What did you notice to lead you to this conclusion? a. Limbs flailing independently b. Open eyes c. Twisting and limbs moving synchronously d. A resting tremor e. Screaming in gibberish

a. Limbs flailing independently- this is found in patients who are experiencing a seizure b. Open eyes- this is found in patients who are experiencing a seizure c. Twisting and limbs moving synchronously- CORRECT- these are characteristics of a pseudoseizure d. A resting tremor- this is not related and can be found in parkinson's patients e. Screaming in gibberish- though something amusing to witness, it is not a characteristic of a pseudoseizure

Mr. Bishop, a 19 year African American male, presents to acute care with a severe TBI following a motorcycle collision. The patient is determined to be an 8 on the Glasgow Coma Scale. Your CI wants to know exactly what determines this severity grade. What do you tell your CI? a. Loss of Consciousness less than 6 hours, abnormal CT or MRI, and Post Traumatic Amnesia lasting less than 6 days determines this score. b. Loss of consciousness less than 25 minutes, a normal CT or MRI, and Post Traumatic Amnesia less than 24 hours determines this score. c. Loss of consciousness greater than 6 hours, Abnormal CT or MRI, and Post traumatic amnesia greater than 7 days determines this score. d. Loss of consciousness greater than 6 hours, Abnormal X-ray, and Post traumatic amnesia greater than 7 days predicts this score. e. Loss of consciousness less than 25 minutes with Abnormal X ray and post-traumatic amnesia greater than 12 days determines this scores.

a. Loss of Consciousness less than 6 hours, abnormal CT or MRI, and Post Traumatic Amnesia lasting less than 6 days determines this score. INCORRECT- This is the classification for a score of 9-12 which is Moderate on the GCS. b. Loss of consciousness less than 25 minutes, a normal CT or MRI, and Post Traumatic Amnesia less than 24 hours determines this score. -INCORRECT- this is the classification for a Mild TBI which scores between 13 and 15. c. Loss of consciousness greater than 6 hours, Abnormal CT or MRI, and Post traumatic amnesia greater than 7 days determines this score.-CORRECT- This is the classification for a Severe TBI with a score < 9. d. Loss of consciousness greater than 6 hours, Abnormal X-ray, and Post traumatic amnesia greater than 7 days predicts this score. -INCORRECT- Although the time components are correct for a Severe TBI, an X-ray will not be used to determine the level of injury. e. Loss of consciousness less than 25 minutes with Abnormal X ray and post-traumatic amnesia greater than 12 days determines this scores. -INCORRECT- Although post-traumatic amnesia greater than 12 days would fall under the severe category on the GCS, loss of consciousness of less than 25 minutes would fall under the Mild TBI category. Furthermore, a standard radiograph would not be used to determine TBI severity.

You are on the hospital floor and are about to meet a stroke patient for the first time. While discussing the case with the attending nurse, you learn that the patient experienced an ischemic stroke. Even before entering the room, what do you know the patient is at an increased risk of exhibiting following an ischemic stroke? a. Lower extremity spasticity b. Memory issues c. Facial sensory deficits d. Vision deficits e. Upper extremity flexion synergies

a. Lower extremity spasticity - potentially, but the hippocampus and cerebellum have the most sensitive neurons to ischemia. b. Memory issues- CORRECT - neurons in the hippocampus, which is the structure involved in memory, are among the most sensitive to ischemic stroke c. Facial sensory deficits - Neurons in the brainstem are most resistant, which includes the trigeminal and facial nerve. d. Vision deficits - potentially, but the hippocampus and cerebellum have the most sensitive neurons to ischemia. e. Upper extremity flexion synergies- potentially, but the hippocampus and cerebellum have the most sensitive neurons to ischemia.

You are treating a 19 year old TBI patient in the inpatient rehab gym at Walter Reed. You have been working with him for 2 weeks now and he has been progressing normally, but today he complains of a rather severe headache that will not go away. Given his past medical history of subarachnoid hemorrhage, you suspect he may be having another one. You suggest the physicians order a CT, and it comes back negative. However, you are still unsure and would like to request further testing, just to be positive. You would order a _______ and are looking for ________. a. Lumbar puncture, elevated PMNs b. Blood sample, elevated WBCs c. Lumbar puncture, elevated proteins and mononuclear cells d. Urine sample, RBCs e. Lumbar puncture, blood and xanthochromic supernatant

a. Lumbar puncture, elevated PMNs - The test is correct, but elevated PMNs would be present in a bacterial infection. b. Blood sample, elevated WBCs - The test is incorrect, and WBCs would not be elevated with a subarachnoid hemorrhage. c. Lumbar puncture, elevated proteins and mononuclear cells - The test is correct, but elevated proteins and mononuclear cells would be present with aseptic meningitis. d. Urine sample, RBCs - The test is incorrect, and blood in your urine would not be present with a subarachnoid hemorrhage. e. Lumbar puncture, blood and xanthochromic supernatant -CORRECT - With a subarachnoid hemorrhage, blood and xanthochromic supernatant will be present in the CSF. CSF is analyzed with a lumbar puncture.

Ms. Reynolds is a 16 year old caucasian female you are treating for whiplash in your outpatient clinic one week post MVA. During treatment, she complains of left sided headache and a strange neck pain and isn't affected by movement of her neck. Upon looking at a mirror, she comments how her left eye seems sunken in and "smaller" in appearance. You recognize the severity of the situation and get her to the ER right away. She gets an MRI angiography and the ER doctor shows you the MRI and says you made a good catch. What exactly did you find? a. MCA occlusion b. ACA stroke c. Internal carotid artery dissection d. Basilar artery occlusion e. PICA dissection

a. MCA occlusion - MCA stroke would present with some unilateral symptoms, but the blood vessel in the MRI is not the middle cerebral artery but the internal carotid artery. b. ACA stroke - ACA stroke would present with more symptoms in lower extremity and some cognitive symptoms. c. Internal carotid artery dissection - CORRECT - ICA dissection has the cervicalgia and unilateral symptoms. (Fred: MCA occlusion would present more dramatically with UE > LE weakness.) d. Basilar artery occlusion - Basilar artery occlusion would have more bilateral symptoms and reduced level of consciousness. e. PICA dissection - PICA dissection would have some impairments involving balance and coordination in addition to ptosis and miosis.

You are a student physical therapist in the UDPT Sports and Orthopedic Clinic and have an evaluation on your schedule for a 36 year old female, Mrs. Campbell, with neck pain. When you enter the room to start your evaluation you notice she has papers disorganized on the treatment table and is dancing around the room. As you collect history, she responds loudly and demonstrates intense emotions when describing her pain. Which mood disorder do you suspect? a. Manic Episode b. Depression c. Anxiety d. Bipolar disorder e. Dysthymia

a. Manic Episode - CORRECT - abnormally active and disorganized b. Depression - associated with depressed mood, changes in eating and sleeping patterns, loss of energy and initiative, low self-esteem, poor concentration, lack of enjoyment of previously pleasurable activities, self-destructive or suicidal thoughts and behavior c. Anxiety - associated with tense posture, restlessness, and fidgeting d. Bipolar disorder - described by alternating periods of mania and depression e. Dysthymia - reports a low mood almost daily over a span of at least two years

You are treating Mrs. Smith for low back pain and have become very close with her. Today she comes in 10 minutes late looking very disheveled and is carrying her 3 month old baby girl. She explains that she is late because her daughter's 3 month check up ran a little late. You notice that her daughter is extremely fussy today and her forehead seems to be swollen compared to normal. When you ask Mrs. Smith she admits that he daughter had been very irritable for the past 2 weeks. You begin to suspect that Mrs. Smith's daughter may have _________ and you would be sure to ask Mrs. Smith if her pediatrician took which measurement at her daughter's 3 month check up? a. Microcephaly, head circumference b. Anhidrosis, weight c. Dysmorphism, height d. Hydrocephalus, head circumference e. A poopy diaper, temperature

a. Microcephaly, head circumference - Although head circumference would be a good measure to determine microcephaly, it is the failure of the brain to grow at an appropriate rate and it's characteristic is a small head, rather than a large head circumference and irritability like with hydrocephalus. b. Anhidrosis, weight - Anhidrosis is lack of sweating, and is not characterized by irritability. Although weight is an important measure for the physician to take, it would not be the most important in diagnosing a patient with this set of symptoms. c. Dysmorphism, height - Dysmorphism is characterized by abnormal development, particularly of facial features. Height is a necessary measurement for children but again it is not the most important in diagnosing a patient with this set of symptoms. d. Hydrocephalus, head circumference - CORRECT Irritability and bulging fontanelles are signs of hyrocephalus, and an important measure for the physician to take would have been head circumference. e. A poopy diaper, temperature - While a poopy diaper may certainly cause a child to be fussy, it does not explain irritability lasting for 2 weeks (unless you are a horrible parent) or the increased head size.

Fifty-nine year old Mr. Jackson is a photographer who suffered a minor stroke several days ago. At the hospital, the doctor told him his CT showed early signs of lack of blood flow to the area of his brain that was affected during his stroke. Mr. Jackson was tired of his wife nagging him to do what the doctor told him so he decided to take advantage of the first snowfall of the year and went out for a drive on the back country roads. Unfortunately, he slammed on his brakes to avoid a stubborn cow and crashed his truck. He was unable to get out of his truck as his doors were stuck. He remained in his truck for three days, during which severe hypothermia set in and his blood pressure experienced severe drops to below 90/50 mmHg. What is true regarding this situation? a. Mr. Jackson is now at risk for a worse outcome regarding his stroke solely because he experienced hypothermia. b. Mr. Jackson is now at risk for a worse outcome regarding his stroke because not only did he have early signs of ischemia, but his blood pressure has dropped below 100/70. c. Mr. Jackson is not at increased risk for a worse outcome regarding his stroke because his BP is only in response to hypothermia. d. Mr. Jackson is only at risk if the cow is mad. e. Mr. Jackson is now at risk for a worse outcome because if he makes it out alive his wife is going to beat him for leaving the house during a winter storm.

a. Mr. Jackson is now at risk for a worse outcome regarding his stroke solely because he experienced hypothermia - hyperthermia, not hypothermia, poses increased risk for worse outcome. b. Mr. Jackson is now at risk for a worse outcome regarding his stroke because not only did he have early signs of ischemia, but his blood pressure has dropped below 100/70 - CORRECT - SBP below 100 and DBP below 70, as well as CT signs of ischemia will prove troublesome for Mr. Jackson's outcome. c. Mr. Jackson is not at increased risk for a worse outcome regarding his stroke because his BP is only in response to hypothermia - his BP and CT findings indicate otherwise. d. Mr. Jackson is only at risk if the cow is mad - the cow is actually thankful Mr. Jackson missed hitting him. e. Mr. Jackson is now at risk for a worse outcome because if he makes it out alive his wife is going to beat him for leaving the house during a winter storm - Mr. Jackson's wife is not that mean.

Mrs. Brown is a 50 year old female experiencing signs and symptoms of a stroke. Mrs. Brown feels she is too young to suffer a stroke so she ignores it. One hour later, her son comes home from school. As a PT student, he recognizes that she is most likely having a stroke and calls 911. As a result of her son's quick action, Mrs. Brown falls within the effective time frame to administer rt-PA but still thinks "you are all nuts" and denies treatment. The doctor administers rt-PA anyway. Some time later, Mrs. Brown is feeling "off" and begins vomiting, her eyes and lips become swollen, and she kicks everyone out because she is feeling tired. What is true regarding this scenario? a. Mrs. Brown is feeling sick because she is having an allergic reaction to the hospital. b. Mrs. Brown is experiencing progression of her stroke symptoms as they can continue to manifest as more brain cells die. c. Mrs. Brown's son slipped her some arsenic when she wasn't looking. d. Mrs. Brown is experiencing complications following rt-PA since she was not an eligible candidate for rt-PA due to her ankle reconstruction surgery 1 month ago. e. Mrs. Brown is experiencing complications following rt-PA including ICH (intracranial hemorrhage) and angioedema.

a. Mrs. Brown is feeling sick because she is having an allergic reaction to the hospital - while hospital food may suck, her symptoms are suggesting something else. b. Mrs. Brown is experiencing progression of her stroke symptoms as they can continue to manifest as more brain cells die - while stroke symptoms can progress over hours/days, these are not typical signs and symptoms of a stroke. c. Mrs. Brown's son slipped her some arsenic when she wasn't looking - her son was the one to take her to the hospital, if he wanted to kill her he would have let her stay at home d. Mrs. Brown is experiencing complications following rt-PA since she was not an eligible candidate for rt-PA due to her ankle reconstruction surgery one month ago - while she is experiencing complications of rt-PA, her surgery date is not within the eligibility cut-off of 14 days or less. e. Mrs. Brown is experiencing complications following rt-PA including ICH (intracranial hemorrhage) and angioedema - CORRECT - Mrs. Brown is experiencing vomiting secondary to ICH and swelling of the eyes and lips is a common presentation of angioedema. Depression of consciousness and feeling of "something wrong" are also signs of ICH. (Fred: 1) Not sure why the doctor administered the tPA without patient permission! 2) Arsenic?!?!? You guys can be more creative than that.)

Excessive sweating can cause dehydration and viscous changes to blood flow, however, I don't think it affects the CSF balance greatly. You are evaluating Mr. Castle, a 26-year-old male who has a script for neck pain and cervicogenic headaches. During the subjective Mr. Castle describes feeling lethargic, vomiting, and experiencing weakness, numbness, and double vision. You recognize these as signs/symptoms of intracranial hypertension and you immediately refer him for further medical work-up. You did not hesitate, because you know compensations for intracranial hypertension include shrinkage of brain tissue through loss of intracellular fluid or reduction of CSF volume via all of the following EXCEPT: a. Narrowing of the ventricular system b. Increased venous absorption of CSF c. Reduction of cerebral blood volume d. Narrowing of the subarachnoid spaces e. Increased excretion of fluids through sweat

a. Narrowing of the ventricular system- this is considered a compensatory mechanism b. Increased venous absorption of CSF- this is considered a compensatory mechanism c. Reduction of cerebral blood volume- this is considered a compensatory mechanism d. Narrowing of the subarachnoid spaces- this is considered a compensatory mechanism e. Increased excretion of fluids through sweat- CORRECT, this is not a method for the reduction of intracranial fluids

You are about to evaluate a 28 year old Caucasian female who was recently admitted to Acute Care due to stroke symptoms. You understand in an examination, the goals are to confirm the presence of a stroke, document degree of deficit, localize the lesion, determine the cause, detect extracranial causes of symptoms, and distinguish stroke from stroke mimics. Due to her young age, you'd like to quickly rule out meningitis, which of the following is NOT a way to rule it out: a. Neck flexion b. Palpation of cervical extensors c. Cervical rotation with a progression of speed in rotation d. Nausea and vomiting e. Complaints of severe headaches with light

a. Neck flexion - neck flexion will cause severe pain with meningitis b. Palpation of cervical extensors- meningitis will cause severe nuchal rigidity c. Cervical rotation with a progression of speed in rotation - cervical rotation will cause a severe headache in people with meningitis d. Nausea and vomiting- CORRECT - nausea and vomiting are common complaints of both stroke and meningitis, therefore it would not be a good tool to use to differentiate e. Complaints of severe headaches with light - meningitis can cause severe photophobia

Arnie Sullivan is a 78 year old African American male, with a PMH significant for MI and HTN. Upon examination, patient was oriented x1 (his name). Patient had diminished touch sensation, vibration and significant weakness in his right lower extremity. Patient also demonstrated very inappropriate behaviors towards the nursing staff, a personality trait abnormal according to his son. Combined you know these symptoms are most likely neurologic in origin. Before going into the patient's room, in the acute care hospital, your CI wants to know what you think her medical diagnosis might be. What is the most likely diagnosis? a. PICA CVA b. Right MCA CVA c. Left MCA CVA d. ACA CVA e. Mid-Basilar Artery Stroke

a. PICA CVA- PICA strokes do not usually show disinhibited or inappropriate behaviors b. Right MCA CVA- decreased weakness is typically seen contralaterally in R MCA c. Left MCA CVA- MCA strokes don't typically present with inappropriate behaviors/disinhibition d. ACA CVA: CORRECT- ACA strokes present with diminished touch sensation, vibration and significant weakness particularly in the contralateral LE. Because this involves the frontal lobe, disinhibition and altered mental status can be seen. e. Mid-Basilar Artery Stroke- quadriparesis is typically seen with mid-basilar artery strokes

Mr. Smith is a seventy-two year old Caucasian male, admitted to the ER secondary to complaints of severe headache, and wife's report of altered mental status. You see Mr. Smith in the acute care hospital with orders for OOB, ambulate with assistance. Review of OT's IE, reveals Mr. Smith's impairments as L hemiparesis and sensory loss, L sided neglect and noted inability of the patient to recognize the socks he was told to put on his feet, consistent with agnosia. Speech Language Pathology was also consulted because the patient was having difficulty swallowing. From your PT education you are confident these are all signs and symptoms of a stroke. What is the most likely medical diagnosis related to these symptoms? a. PICA CVA b. Right MCA CVA c. Left MCA CVA d. ACA CVA e. Mid-Basilar Artery CVA

a. PICA CVA: Although PICA CVA's can present with dysphagia and weakness, they don't typically have agnosia. b. Right MCA CVA: CORRECT Right MCA's present with L hemiparesis and sensory loss, L sided neglect, dysphagia and agnosia. c. Left MCA CVA: MCA infarcts typically present with contralateral weakness and sensory loss, rather than ipsilateral weakness and sensory loss d. ACA CVA: although symptoms of ACA are typically of contralateral weakness, sensory loss, and gait apraxia, they typically do not include dysphagia of agnosia. e. Mid-Basilar Artery CVA- mid basilar artery strokes typically present with quadriparesis, not contralateral

Mr. Clooney is a 45 y/o male you are visiting at the hospital s/p TBI. Before entering the room, you check his chart and see the following: a CT of a large epidural hematoma, a progress note that stated that the pt experienced a 4-hour coma, and a GCS of 10, as per the neurologist's note. Knowing this information, you can conclude that the patient has experienced which grade of a TBI? a. Severe b. Moderate c. Mild d. Partial e. More information needed

a. Severe - Although there is an abnormal CT, the other criteria for a "severe" are LOC >6 hrs, GCS <9m and PTA >7 days b. Moderate - CORRECT- The criteria for a "moderate" TBI are LOC <6 hrs, abnormal CT/MRI, GCS 9-12, and PTA <7 days c. Mild - A "mild" TBI is classified as LOC <30 min, normal CT or MRI, GCS 13-15, and PTA <24 hrs d. Partial - The severity gradings of TBIs are severe, moderate, or mild. e. More information needed - Enough information is provided to conclude a moderate TBI.

Patient is a 65 year old African American male, presenting to the ER because wife reports the patient seemed "extremely out of it" and was unable to get out of bed. Upon chart review patient presents with quadriparesis and vertical nystagmus with eye elevation. You know vertical nystagmus in particular is only caused by a few pathologies. You have been consulted to evaluate this patient for PT in the acute care setting. What is the most likely cause of these symptoms? a. PICA CVA b. Right MCA CVA c. Left MCA CVA d. ACA CVA e. Mid-Basilar Artery CVA

a. PICA CVA: Although PICA patients can present with nystagmus and other eye impairments, quadriparesis is a symptom typical of a mid-basilar artery stroke b. R MCA CVA- MCA infarcts typically show contralateral hemiparesis, rather than quadriparesis c. L MCA CVA- MCA infarcts typically show contralateral hemiparesis, rather than quadriparesis d. ACA CVA- symptoms of ACA are typically of contralateral weakness, sensory loss, gait apraxia and disinhibition e. Mid-Basilar Artery Stroke: CORRECT- Mid-Basilar Artery CVA's can be determined because of the quadriparesis and vertical nystagmus present. These symptoms are typical of a Mid-Basilar Artery stroke.

Mr. Fleisner is a 92-year-old black male presenting to PT 2 weeks s/p hemorrhagic stroke. While taking the subjective, the patient recalls the MD stated that his MRI showed the majority of damage to be in the "watershed" areas. Because of your training you immediately recognize that the MD is most likely referring to: a. PICA distribution b. the area between ASA/PSA c. Pontine arteries distribution d. the area between MCA/PCA e. the gyri

a. PICA distribution- watershed areas refer to areas between two arteries b. the area between ASA/PSA- this answer is incorrect because these arteries are found in the spinal cord c. Pontine arteries distribution- watershed areas refer to areas between two major arteries d. the area between MCA/PCA- CORRECT- this is a major watershed area in the brain and a likely place for blood to be found with a hemorrhagic stroke e. the gyri- this answer does not make sense with the the question being asked.

You are in an outpatient clinic and in walks 80 y/o Mr. Brooks. You observe that his gait is atypical: he is walking with a stooped posture, demonstrating slow initiation of movements, and short shuffling steps. At the beginning of your exam, when he first sits in your office, you notice Mr. Brooks' right hand is fluttering in his lap. You hypothesize that he may have_________ which causes defects to the , which, in turn, causes a lack of the neurotransmitter _________. a) Parkinson's; substantia nigra; dopamine b) Cerebral Palsy; substantia nigra; acetylcholine c) Parkinson's; pons; dopamine d) Huntington's; DNA; GABA e) Nothing it's just old age

a. Parkinson's; substantia nigra; dopamine - CORRECT signs of Parkinson's gait and resting tremor; Parkinson's affects the substantia nigra causing cell death, which causes underproduction of dopamine b. Cerebral Palsy; substantia nigra; acetylcholine- CP is a non-progressive disease seen early in life, in which you typically see spasticity; reduced acetylcholine is characteristic of Alzheimer's c. Parkinson's; pons; dopamine- Parkinson's affects the substantia nigra d. Huntington's; DNA; - Huntington's is characterized by chorea; low levels of GABA are associated with depression, Bipolar Disorder, and other personality disorders, like anhedonia e. Nothing it's just old age- these are signs of Parkinson's

Mrs. Summers, a 76 year old female, presents to the NOA clinic for treatment of her cervical neck pain. Upon evaluation you suspect the patient is depressed, so you ask her, "Have you felt sad, lonely, or depressed lately?" The following signs and symptoms during the evaluation lead you to ask this question EXCEPT: a. Poor concentration b. Memory loss c. Changes in eating/sleeping patterns d. Slumped posture e. Low self-esteem

a. Poor concentration - this is true of depression b. Memory loss - CORRECT - this is not a symptom of depression (Fred: Depression CAN cause memory loss) c. Changes in eating/sleeping patterns - this is true of depression d. Slumped posture - this is true of depression e. Low self-esteem - this is true of depression

You are a PT treating two patients with paresthesias in the exact same distribution pattern. Both patients have some form of sensory loss below the level of T5, throughout the trunk and in bilateral lower extremities. Patient A has loss of pain, temperature, and light touch, while vibration and joint position are intact. Patient B has loss of vibration and joint position, while pain, temperature and light touch are intact. Having a firm understanding of spinal tracts, you know that patient A has a/an _____________ and patient B has a/an ______________. a. Posterior Column lesion; Hemisection of the cord b. Central Cord lesion; Anterior Spinal Syndrome c. Thalamic lesion; Hemisection of the cord d. Anterior Spinal Syndrome; Posterior Column lesion e. Brain stem lesion; Thalamic lesion

a. Posterior Column lesion; Hemisection of the cord - would result in loss of light touch, pain and temperature below level of lesion; & loss of light touch, pain and temperature below the level of the lesion on the contralateral side b. Central Cord lesion; Anterior Spinal Syndrome - would result in pain and temperature below the level of the lesion; & loss of pain, temperature, and light touch below the level of the lesion c. Thalamic lesion; Hemisection of the cord - would result in ipsilateral loss of vibration, joint position, light touch, pain, and temperature; & loss of light touch, pain and temperature below the level of the lesion on the contralateral side d. Anterior Spinal Syndrome; Posterior Column lesion - CORRECT The sensory loss distribution for Anterior Spinal Syndrome is loss of pain, temperature, and light touch, with vibration and joint position intact and a Posterior Column lesion would result in loss of vibration and joint position, with pain, temperature and light touch intact. e. Brain stem lesion; Thalamic lesion - would result in loss of light touch, pain, and temperature on the ipsilateral face and contralateral body; & ipsilateral loss of vibration, joint position, light touch, pain, and temperature

A. Love, a former English professor, is in your outpatient clinic to address L knee pain. You ask him to fill out a KOS form because it is time for a re-eval. His wife is with him and said she had to fill out the form for him because he doesn't seem to be able to read or write since yesterday. She's also very upset because he isn't paying attention to her (especially when she is standing to his L) and he has forgotten her name multiple times. You immediately tell her to call 911 and get him to the hospital. She thinks you cannot be serious and that her husband is probably joking around with her. You tell her you are very serious because he may have experienced a(n): a. Posterior cerebral artery infarct b. Basal ganglia hemorrhage c. Frontal lobe contusion d. Anterior-inferior temporal contusion e. Unilateral superior mesial frontal lobe damage

a. Posterior cerebral artery infarct - CORRECT. Alexia, agraphia and neglect are symptoms associated with an infarct of this nature. b. Basal ganglia hemorrhage - These are associated with more movement related symptoms. c. Frontal lobe contusion - Contusions here are associated with personality changes, impulsivity. While lack of attention is a symptom, alexia and agraphia are not. d. Anterior-inferior temporal contusion - Agnosia, psychosis and altered sexual behavior are symptoms with these types of contusions. e. Unilateral superior mesial frontal lobe damage - Poor initiation, reduced spontaneity and expression are common symptoms of this.

You are working in an outpatient PT clinic and you notice your patient, Mr. Greenandyellowbackpack, is on antihypertensives. He reports that he has not taken his blood pressure medication today. You decide not to allow him to participate in any exercise that will raise his blood pressure more than 20 mmHg because you are concerned about a spontaneous hemorrhagic stroke. Where is the most common location for a spontaneous hemorrhagic stroke and what structures does this supply blood to? a. Posterior spinal arteries below the level of the internal arcuate fibers; dorsal column medial lemniscal system b. Lenticulostriate arteries; deep grey nuclei c. Central retinal arteries; the retina. d. PICA; posterior and inferior aspects of the cerebellum e. Pontine perforating arteries; the Pons.

a. Posterior spinal arteries below the level of the internal arcuate fibers; dorsal column medial lemniscal system INCORRECT. Correct pairing of artery with structure supplied; this is not the most common location for spontaneous hemorrhagic stroke. b. Lenticulostriate arteries; deep grey nuclei CORRECT. This is the most common location of spontaneous hemorrhagic stroke and is paired with the correct structures. c. Central retinal arteries; the retina. INCORRECT. Correct pairing of artery with structure supplied; this is not the most common location for spontaneous hemorrhagic stroke. d. PICA; posterior and inferior aspects of the cerebellum INCORRECT. Correct pairing of artery with structure supplied; this is not the most common location for spontaneous hemorrhagic stroke. e. Pontine perforating arteries; the Pons. INCORRECT. Correct pairing of artery with structure supplied; this is not the most common location for spontaneous hemorrhagic stroke.

You are in an outpatient ortho PT setting, treating your 30 y/o patient who is in her third trimester of pregnancy. You are treating her LBP, but today she is complaining of a new pain in her right calf. Being the intelligent UD grad that you are, you investigate further, decide to hold PT that day, and refer her to the ER because: a. Pregnancy is a "disease state" that prevents women from contracting diseases and conditions which in turn protects the fetus b. There is decreased risk for many specific conditions such as eclampsia, choriocarcinoma, and amniotic fluid embolism c. Women are hypercoagulable during pregnancy and therefore are at increased risk for DVT. Her symptoms and her score on the Wells and Colleagues Clinical Decision Rule suggest she may have a DVT d. Pregnant women are at increased risk for venous strokes, specifically e. There is a decreased risk for postpartum cerebral angiopathy and peripartum cardiomyopathy

a. Pregnancy is a "disease state" that prevents women from contracting diseases and conditions which in turn protects the fetus- pregnancy alters the normal body processes and puts the mother at increased risk for many conditions b. There is decreased risk for many specific conditions such as eclampsia, choriocarcinoma, and amniotic fluid embolism- there is an increased risk for these conditions c. Women are hypercoagulable during pregnancy and therefore are at increased risk for DVT. Her symptoms and her score on the Wells and Colleagues Clinical Decision Rule suggest she may have a DVT- CORRECT- you would be concerned that your patient may have developed a DVT d. Pregnant women are at increased risk for venous strokes, specifically- pregnant women are at increased risk for arterial strokes e. There is a decreased risk for postpartum cerebral angiopathy and peripartum cardiomyopathy- there is an increased risk for these conditions

You are evaluating a 62-year-old woman, who is recovering from a stroke. She complains that at times she can't see out of her R eye for a few minutes, and then it gets better. She remarks that she has looked in the mirror when this happens and her right eye is not actually closed. She tells you she is a biology professor at UD and is curious about what's going on in her eye, and what the condition is called. You tell her the condition is known as __________, and it is caused by ________. a. Ptosis; oculomotor nerve palsy b. Amaurosis fugax; a large ischemic stroke, likely to her right ophthalmic or retinal artery c. Amaurosis fugax; a temporary lack of blood flow to the retina from the ophthalmic artery or the retinal artery d. Bell's Palsy; various causes, all of which involve some pathology to cranial nerve VII e. Ptosis; temporary compression of the optic nerve

a. Ptosis; oculomotor nerve palsy → INCORRECT. Ptosis is caused by oculomotor nerve palsy, but will not cause loss of vision as your patient has described, it may cause loss of vision due to drooping eyelid or loss of motor control of the eyeball, and your patient has noted that her eye is not actually closed during her episodes of vision loss. b. Amaurosis fugax; a large ischemic stroke, likely to her right opthalmic artery → INCORRECT. Amaurosis fugax is a form of TIA, not stroke. c. Amaurosis fugax; a temporary lack of blood flow to the retina from the opthalmic artery or the retinal artery → CORRECT. Amaurosis fugax is a form of TIA, usually affecting one eye by occluding the ophthalmic or retinal artery. d. Bell's Palsy; various causes, all of which involve some pathology to cranial nerve VII → INCORRECT. Bell's Palsy does affect cranial nerve VII, but this nerve has little influence on the function of the eye. e. Ptosis; temporary compression of the optic nerve. → INCORRECT. Ptosis would cause the eyelid to droop, but is caused by pathology to the oculomotor nerve, not the optic nerve, and your patient noted that her eye is not closed, but rather that her vision is occluded.

Mr. Lum is a 63 y/o male with a spinal cord injury whom you are treating in the NOA clinic for neck pain. During the evaluation you establish that despite being painful, Mr. Lum's cervical ROM is within normal limits, however, he appears to have difficulty coordinating his head and eye movements during VOR reflex testing. Assuming this discord is the result of the spinal cord lesion, which tracts are most likely affected? a. Reticulospinal b. Spinocerebellar c. Vestibulospinal d. Rubrospinal e. Medial Longitudinal Fasciculus

a. Reticulospinal - (incorrect - is mainly involved in locomotion and postural control) b. Spinocerebellar - (incorrect - involved with movement control, but not specific to head and eye movement coordination) c. Vestibulospinal - (incorrect - provides control of anti-gravity muscles) d. Rubrospinal - (incorrect - primarily facilitates flexion in the upper extremities) e. Medial Longitudinal Fasciculus - (CORRECT - coordinates head and eye movements and intermingles with the tectospinal tract)

This is a content error. The answer should be C. Please rewrite the question and resubmit with the next set. Another member of the group should have picked this up AND whichever group was doing the editing should have also mentioned something. Mrs. Keffer is a 60 year old teacher you are gait training in outpatient rehab three months after an ischemic CVA. She has a history of heart disease and previous episodes of thrombi originating from her heart. What is the most likely chamber for these blood clots to originate from and which imaging technique would be most appropriate to assess her condition? a. Right Atrium, Transesophageal echocardiography (TEE) b. Right Atrium, Transthoracic echocardiography (TTE) c. Left Atrium, Transesophageal echocardiography (TEE) d. Left Atrium, Transesophageal echocardiography (TTE) e. Coronary Sulcus, X-ray

a. Right Atrium, Transesophageal echocardiography - INCORRECT - The left atrium has the most instances of blood clot formations and it is the most posteriorly located portion of the heart abutting the esophagus and TEE would get the best imaging of the left Atrium. b. Right Atrium, Transthoracic echocardiography - The left atrium has the most instances of blood clot formations and it is the most posteriorly located portion of the heart abutting the esophagus but TTE would have to go thru other chambers before it can a clear image of the left atrium. c. Left Atrium, Transesophageal echocardiography - The Left atrium is the origin of most clots from the heart and TEE would get the best imaging of the left Atrium. d. Left Atrium, Transesophageal echocardiography - The Left atrium is the origin of most clots from the heart and TTE would get the best imaging of the left Atrium. e. Coronary Sulcus, Diffusion Tensor Imaging - The coronary sulcus is not the origin of most clots from the heart and X-ray would not give you very good imaging of soft tissue, X-rays are better at imaging hard tissues such as bone.

Mr. Cooper is a 65 years old male who presents to your inpatient rehabilitation center three days s/p acute cerebral ischemia. He has received thrombolytic therapy immediately after the event; however, upon reviewing his medical chart (including CT scan) and assessing his vitals, you conclude the patient has a poor prognosis due to: a. SBP = 140, DBP = 90 b. Low blood glucose level c. Hypothermia lasting > 2 days d. SBP = 120, DBP =70 e. Early CT evidence of infarction

a. SBP = 140, DBP = 90 - worse outcome is expected with low blood pressure (SBP <100 DBP <70 mmHg) b. Low blood glucose level - worse outcome is expected with hyperglycemia due to reduced reperfusion c. Hypothermia lasting > 2 days - worse outcome is expected with hyperthermia first 12-24 hours up to several days after stroke due to failure of thrombolysis d. SBP = 120, DBP = 70 - worse outcome is expected if SBP falls below 100 and DBP below 70 mmHg e. Early CT evidence of infarction - CORRECT - this is an indication for increased tendency for hemorrhagic transformation after thrombolytic (Fred: I think it has to do with the size of the ischemia rather than how early it presents on CT.)

Your patient is a 27 year old white male admitted to the TBI wing of the hospital after being in a MVA. You are assessing his ROM and muscle strength when he suddenly appears to be convulsing. You quickly realize he is not having a pseudo-seizure, and you clear the area as best you can and call for a nurse to assist you. You take a look at his chart to assess his epilepsy risk factors. All of the following are risk factors EXCEPT: a. Skull Fractures b. Intracranial hemorrhage c. Increased severity of injury d. History of chronic caffeine use e. Wounds that penetrate the brain

a. Skull Fractures-INCORRECT - skull fractures are traumatic to the brain and can cause abnormal neural signaling, increasing one's risk for epilepsy. b. Intracranial hemorrhage-INCORRECT - intracranial hemorrhaging also increases the risk of epilepsy due to a disruption of the brain's normal blood flow, affecting normal brain activity. c. Increased severity of injury-INCORRECT - the more severe an injury the more likely it will disturb normal brain activity, increasing the likelihood of causing epilepsy. d. History of chronic caffeine use- CORRECT - chronic caffeine use has not been shown to be a risk factor for epilepsy. Otherwise most PT students would increase their chances for epilepsy. e. Wounds that penetrate the brain-INCORRECT - wounds directly penetrating the brain cause increased abnormal neural activity and signaling, which will increase the risk for epilepsy.

While reviewing Mr. Fumar's chart in the hospital, an 86 year old male s/p right ischemic stroke, you notice he is taking Bupropion. For what is he taking this medication? a. Spasticity and Smoking b. Depression and Smoking c. Seizures and Depression d. Spasticity and Seizures e. Smoking and Seizures

a. Spasticity and Smoking- this medication is not used to decrease spasticity but is used for smoking cessation b. Depression and Smoking- CORRECT- this medication is used as an anti-depressant and for smoking cessation c. Seizures and Depression- this medication is not to prevent seizures but is used to treat depression d. Spasticity and Seizures- this medication is not used to treat either spasticity or seizures e. Smoking and Seizures- this medication is used for smoking cessation but not used to prevent seizures

You are treating Mrs. Hunt, a 74 year old female s/p left CVA, in your outpatient physical therapy clinic. Upon ROM testing, you notice spasticity in her right plantarflexors. All of the following are treatments for spasticity EXCEPT: a. Splinting b. Tendon lengthening c. Banzel d. Botulinum toxin e. Proper positioning

a. Splinting- this is a treatment for spasticity b. Tendon lengthening- this is a treatment for spasticity c. Banzel-CORRECT- this is a treatment for epilepsy, not spasticity d. Botulinum toxin- this is a treatment for spasticity e. Proper positioning- this is a treatment for spasticity

While working in your outpatient physical therapy clinic, you receive a prescription for an 87 year old female, Mrs. Mummert, that states "evaluate and treat for balance and gait deviations." When you greet your patient in the waiting room you notice she has signs and symptoms consistent with Parkinson's syndrome. Which of the following statements is NOT true about Parkinson's syndrome? a. Parkinson's patients lack dopamine b. A lack of dopamine enables the striatum to discharge excessively, which results in bradykinesia c. Patients present with an increase (or decreased depending on balance of dopamine and neuronal activity) in motor movements d. Patients display increased motor tone to passive movements in all directions e. A resting tremor is usually present

a. Substania nigra has a damping effect on the striatum via dopamine - this is true of Parkinson's syndrome b. The lack of dopamine enables the striatum to discharge excessively - this is true of Parkinson's syndrome c. Patients present with an increase in motor movements - CORRECT - the patient presents with decreased motor movements or decreased depending on balance of dopamine and neuronal activity d. Increased motor tone to passive movements in all directions - this is true of Parkinson's syndrome e. A resting tremor is usually present - this is true of Parkinson's syndrome

While working in an outpatient PT clinic your patient, Mrs. Blueandredtilecarpet, reports she has not taken her blood pressure lowering medications today. She is a 'frequent flyer' at your PT clinic, and you know that she has a history of two subarachnoid hemorrhagic strokes in the last decade. Which of the following is NOT a common (more than 1/3 of subarachnoid hemorrhagic strokes) clinical presentation of SAH? a. Sudden explosive headache b. Vomiting c. A period of unresponsiveness d. Sudden increased excitability and speaking volume e. Focal neurologic signs

a. Sudden explosive headache INCORRECT. Patients often complain of 'the worst headache of their life', and it has a sudden onset, often referred to as thunderclap headaches. b. Vomiting INCORRECT. Many patients do vomit during acute subarachnoid hemorrhagic strokes c. A period of unresponsiveness INCORRECT. Many patients do have a period of unresponsiveness. d. Sudden Increased excitability and speaking volume CORRECT. Patients generally do not demonstrate increased excitability and speaking volume, but rather the opposite. e. Focal neurologic signs INCORRECT. Patients will probably have some focal neurologic signs, as this is affecting their neural tissue.

Charlie Mitchell is a 21 y.o male suffering from an acute TBI due to falling down the stairs at his fraternity house. The injury lead to significant edema in the cranium. From afar you notice that Charlie is being instructed to hyperventilate in an effort to "suck in the edema". Why is acute management of edema associated with a TBI extremely important? a. Swelling will likely lead to decreased ICP b. Edema typically leads to shearing of the grey matter from white matter c. Edema leads to decreased intracranial temperature and therefore cell death d. Swelling in the brain can pinch off vessels and cause ischemia e. This treatment is crazy, edema in the brain is not a problem.

a. Swelling will likely lead to decreased ICP - swelling will likely lead to increased ICP b. Edema typically leads shearing of the grey matter from white matter- this is usually due to force and not swelling c. Edema leads to decreased intracranial temperature and therefore cell death- edema causes cell death due to ischemia and not significant drop in temperature d. Swelling in the brain can pinch of vessels and cause ischemia- CORRECT- ischemia is a major concern with acute management e. This treatment is crazy, edema in the brain is not a problem- if only this were true, treatment would be much easier

You are currently treating a 65 year old Caucasian male in your outpatient clinic who suffered a stroke six months ago. Shortly after beginning his warm-up on the treadmill, he reports a sudden, severe headache, slurred speech, and nausea. You get him off the treadmill and tell your aide to call 911. You understand there is a high possibility of another stroke, but there are also other possibilities. Which of the following is NOT a likely reason for your patient's signs and symptoms? a. Systemic infection b. Hypertensive encephalopathy c. Meningitis d. Brain abscess e. Anemia deficiency

a. Systemic infection-Systemic infection can decrease one's neurological reserve, causing a presentation of acute stroke. b. Hypertensive encephalopathy- Hypertensive encephalopathy can affect the watershed areas of the brain, causing a presentation of an acute stroke. c. Meningitis- Meningitis may cause altered mental status and nausea. d. Brain abscess- Brain abscess can also cause an altered mental status and a headache among many other things. e. Anemia deficiency- CORRECT - anemia can cause fatigue, a rapid heart rate, pallor of the skin and cramps. Headaches can occur as well, but slurred speech and nausea are not common symptoms of an anemia deficiency.

You are taking a subjective history on Mr. Martin, a 35 year old male. He reports a sudden feeling of weakness in his right upper extremity that caused him to drop his briefcase walking from his car to his office. Weakness developing over minutes may be due to: a. TIA, stroke, tumor b. Migraine, seizure, tumor c. Tumor, seizure, brain abscess d. Stroke, migraine, seizure e. Brain abscess, TIA, migraine

a. TIA, stroke, tumor - TIA: abrupt onset, stroke is correct, tumor: weeks to months b. Migraine, seizure, tumor - migraine is correct, seizure is correct, tumor: weeks to months c. Tumor, seizure, brain abscess - tumor: weeks to months, seizure is correct, brain abscess: days d. Stroke, migraine, seizure - CORRECT - all develop over minutes e. Brain abscess, TIA, migraine - brain abscess: days, TIA: abrupt onset, migraine is correct

Mr. Jane is a 54-year-old male in your hospital who recently had a stroke. His chart states he is a two person assist for most activities, so you seek out an aide. In addition, you read he has recently been experiencing shoulder pain. Before you enter his room with the aide, you explain... a. The patient must have been a pitcher when he was younger; it's the only plausible explanation for his shoulder pain. b. Shoulder pain is rare in stroke victims; despite his denials, he must have injured his shoulder during a fall, as this is the only logical explanation for his shoulder pain c. The shoulder pain is the least of your problems right now and should be ignored d. The rotator cuff is vulnerable in the flaccid period after stroke and must be protected during transfers, positioning, and wheelchair seating e. Shoulder pain onset after stroke is always immediate, so the patient must be faking

a. The patient must have been a pitcher when he was younger; it's the only plausible explanation for his shoulder pain.- Though he may have been a pitcher, this is not the only explanation for his shoulder pain- 80% of stroke patients have shoulder pain b. Shoulder pain is rare in stroke victims; despite his denials, he must have injured his shoulder during a fall, as this is the only logical explanation for his shoulder pain- Shoulder pain is common post-stroke (80% incidence) c. The shoulder pain is the least of your problems right now and should be ignored- this is incorrect- care must be taken to protect the shoulder during this phase d. The rotator cuff is vulnerable in the flaccid period after stroke and must be protected during transfers, positioning, and wheelchair seating.- CORRECT e. Shoulder pain onset after stroke is always immediate, so the patient must be faking- incorrect- onset is often delayed

Mr. Pitt is a patient you are seeing in an L.A. acute care hospital s/p TBI 6 days ago from a motorcycle accident. Before entering the room, you want to check his chart for current treatments. You see that his ICP is currently at 25 mmHg. Knowing this information, which of the following WOULD you possibly see in the chart? a. The use of barbiturates b. The use of sedatives c. A recent decompressive craniotomy d. The use of hyperventilation e. The use of antidiuretics

a. The use of sedatives - Sedatives are not found to improve mortality with acute increased ICP. b. The use of barbiturates - Barbiturates are a type of sedative. c. A recent decompressive craniotomy - Decompressive craniotomies, like sedatives, are not used for acute increases in ICP. d. The use of hyperventilation - CORRECT. Hyperventilation is recommended for acute management of TBIs with increased ICP. e. Antidiuretics - Incorrect. Diuretic medications as well as incorporating hyperventilation are suggested for acute TBI

You are starting your first job as an acute care PT at UD Hospital. You are doing a chart review for a SCI patient that you are seeing for the first time today. You notice in his chart that he has symptomatic hypercalcemia, noting that he has been experiencing nausea, dehydration, and overall muscle weakness. As a knowledgeable graduate, you know that symptomatic hypercalcemia means: a. There is an increased amount of sodium concentration in his body. b. Bone is beginning to form in places where it should not be, such as in his soft tissue. c. His blood sugar is too low, causing sweating, trembling, and a fast heartbeat. d. His bone is demineralizing very quickly, which is leading to too much calcium in his bloodstream e. He has an iodine deficiency, which is causing his thyroid gland to produce decreased amounts of TSH.

a. There is an increased amount of sodium concentration in his body. - This is hypernatremia. b. Bone is beginning to form in places where it should not be, such as in his soft tissue. - This is heterotopic ossification. c. His blood sugar is too low, causing sweating, trembling, and a fast heartbeat. - This is hypoglycemia. d. His bone is demineralizing very quickly, which is leading to too much calcium in his bloodstream. - CORRECT - this is the definition of symptomatic hypercalcemia. e. He has an iodine deficiency, which is causing his thyroid gland to produce decreased amounts of TSH. - This is referring to hypothyroidism.

You are working in an acute care hospital and your patient, Mr. Rolliechair, is affected by amyloid angiopathy. Which of the following is NOT true about amyloid plaques? a. They typically affect persons over 70 years of age b. They can affect multiple organs in the body c. All patients with deposition of amyloid plaques in the brain demonstrate progressive dementia. d. Amyloid plaques have been correlated with neurofibrillary tangles and alzheimer's or dementia e. The Camelford incident launched an investigation into the effects of aluminum on deposition of amyloid plaques in the brain

a. They typically affect persons over 70 years of age INCORRECT. Amyloid plaque pathology in the brain is associated with people over 70 years of age b. They can affect multiple organs in the body INCORRECT. Amyloid plaques are not exclusive to the brain. c. All patients with deposition of amyloid plaques in the brain demonstrate progressive dementia. CORRECT. Only ■ of patients with deposition of amyloid plaques in the brain demonstrate progressive dementia. d. Amyloid plaques have been correlated with neurofibrillary tangles and alzheimer's or dementia. INCORRECT - amyloid plaques have been correlated with neurofibrillary tangles and alzheimer's/dementia, although no conclusive causative link has been made. e. The Camelford incident launched an investigation into the effects of aluminum on deposition of amyloid plaques in the brain INCORRECT. The Camelford water pollution incident DID result in British government funded research into the effects of aluminum on deposition of amyloid plaques in the brain.

You are treating Mrs. Schutto, an 85-year-old female s/p stroke, in your acute rehab hospital. Before entering her room, you look through her chart and note she has dementia and aphasia. When you help Mrs. Schutto sit up, you notice her bedding is damp and opt to utilize this opportunity to work with the patient on bed mobility as you change the sheets. Meanwhile, you remember all of the following are true regarding urinary incontinence in stroke victims EXCEPT: a. Urinary incontinence incidence is very rare, <10%, in the early recovery period post-stroke b. Urinary incontinence is correlated with stroke size c. Urinary incontinence can be caused by a hypotonic bladder d. Urinary incontinence usually improves over time e. Urinary incontinence can be a problem due to the patient's inability to communicate

a. Urinary incontinence incidence is very rare, <10%, in the early recovery period post-stroke- CORRECT - this statement is not true; it occurs 38-60% b. Urinary incontinence is correlated with stroke size- this statement is true c. Urinary incontinence can be caused by a hypotonic bladder- this statement is true d. Urinary incontinence usually improves over time- this statement is true e. Urinary incontinence can be a problem due to the patient's inability to communicate- this statement is true especially in patients with dementia and/or aphasia secondary to inability to communicate need to void

Mrs. Hawkins is a 75-year-old female you are seeing in NOA clinic following a pontine stroke. With your extensive knowledge of neuroanatomy and neurology, you want to check CN ___ because she is experiencing some difficulties with her hearing in one ear. You complete the Weber test and find lateralization to the good ear, which you know you get with ________ hearing loss. a. VIII; conductive b. X; sensorineural c. VIII; sensorineural d. X; conductive e. IX; conductive

a. VIII; conductive - you get lateralization to deaf ear with conductive hearing loss b. X; sensorineural - vagus nerve innervates muscles of mouth, trachea, esophagus, heart, stomach and small intestines. c. VIII; sensorineural - CORRECT - sound is heard in good ear (through air) with sensorineural hearing loss d. X; conductive - vagus nerve innervates muscles of mouth, trachea, esophagus, heart, stomach and small intestines and you get lateralization to deaf ear with conductive hearing loss e. IX; conductive - glossopharyngeal nerve does not have anything to do with hearing and you get lateralization to deaf ear with conductive hearing loss

Johan is a 58 year old, African American male being treated in Acute Care for CHF. You enter the room to begin therapy and notice he has drooping of his right eyelid, which is unusual. You take a closer look at his pupils and notice they are constricted. The patient also has decreased pain sensation on the right side of his face and left side of his body. You ask him how he feels and he reports, in a hoarse voice, he's been having trouble swallowing. His signs and symptoms are consistent with what syndrome? a. Wallenberg Syndrome b. Dejerine's Syndrome c. Pon's-Foville Syndrome d. Pon's-Millard-Gubler Syndrome e. Weber's Syndrome

a. Wallenberg Syndrome- CORRECT- This presents with decreased pain and temperature sensation to the ipsilateral face and contralateral body, dysphagia, anhydrosis, miosis, and ptosis. (Fred: With Pons-Fovile, don't forget the ipsilateral LMN presentation of the facial nerve.) b. Dejerine's Syndrome-this presents with contralateral decrease in vibration and proprioceptive sense and ipsilateral CN XII palsy c. Pon's-Foville Syndrome- Presents with ipsilateral gaze paralysis and contralateral hemiparesis. d. Pon's-Millard-Gubler Syndrome- Presents with contralateral hemiparesis. e. Weber's Syndrome- Presents with contralateral hemiparesis and ipsilateral CN III palsy

After you have gathered the objective information from your patient including: Right CN III palsy and Left hemiparesis, you decide to call the MD because you suspect a stroke. The MD performs a work up and diagnoses her with a Right brainstem stroke of the medial midbrain. What is the name of this syndrome and which vessels contributed to this? a. Wallenberg syndrome; PCA perforators b. Weber's syndrome; PCA perforators c. Benedikt's syndrome; PICA d. Weber's syndrome; PICA e. Wallenberg syndrome; PICA

a. Wallenberg syndrome; PCA perforators- Wallenberg Syndrome typically occurs in the lateral medulla. The PICA is the main vascular contributor. Symptoms would include ipsilateral hemi-ataxia rather than contralateral deficits as noted in Weber's syndrome. The patient may also present with dysphagia, hoarseness and ipsilateral palatal weakness, along with decreases in pain and temperature sensation of the ipsilateral facial and contralateral body. b. Weber's syndrome; PCA perforators- CORRECT- The PCA perforators are the main vessels affected with Weber's syndrome. Patients may present with deficits including dilation of the eye, ipsilateral CN III palsy and contralateral hemiparesis c. Benedikt's syndrome; PICA- The PCA perforators are the main vessels implicated in Benedikt's syndrome. Deficits linked to Benedikt's Syndrome may present similar to Weber's syndrome with ipsilateral CNIII palsy and dilation of the eye however those suffering from Benedikt's syndrome will present with contralateral involuntary movements rather than hemiparesis. d. Weber's syndrome; PICA- The syndrome is correct however the PCA perforators are the vessels associated. e. Wallenberg syndrome; PICA- The PICA is the major culprit contributing to Wallenberg syndrome, however, Wallenberg syndrome would have a different presentation than the patient described above. Please see answer (a) for the deficits involved with Wallenberg syndrome.

You are an acute care PT treating a 26 year old male TBI patient who was recently in a motorcycle accident. The collision was head on, so before performing any kind of a cervical assessment, you are sure to review the below imaging test (Fig 1.). You are relieved to see no fractures present, as well as all three anatomical lines (A, B, C) intact. What image is displayed in Fig. 1 and what are the names of lines A, B, and C in order. a. X-ray; A = anterior spinal line; B = posterior spinal line; C = spinolaminar line b. CT; A = frontal spinal line; B = mid spinal line; C = dorsal spinal line c. X-ray; A = spinolaminar line; B = posterior spinal line; C = anterior spinal line d. CT; A = anterior spinal line; B = medial spinal line; C = posterior line e. X-ray; A = pre-spinous line, B = central spinal line, C = posterior spinal lines

a. X-ray; A = anterior spinal line; B = posterior spinal line; C = spinolaminar line - CORRECT- This is the correct test and correct order of anatomical landmarks b. CT; A = frontal spinal line; B = mid spinal line; C = dorsal spinal line - Incorrect test; correct anatomical lines c. X-ray; A = spinolaminar line; B = posterior spinal line; C = anterior spinal line - Correct test; incorrect anatomical line order d. CT; A = anterior spinal line; B = medial spinal line; C = posterior line - Incorrect test; correct anatomical lines e. X-ray; A = pre-spinous line, B = central spinal line, C = posterior spinal lines - Correct test; incorrect anatomical terms

While treating Mr. Vance, a 63 year old male 3 days s/p left ischemic stroke, in the acute care hospital you notice his BP pre-treatment is 190/100. While performing light therapeutic exercise, you notice his BP on the monitor is 202/98. Should you be concerned? a. Yes, HTN is bad in the acute stage following stroke b. Yes, this increase in blood pressure is an abnormal response to light-moderate activity c. No, this is not hypertensive d. No, you want HTN in the acute stage to allow blood flow to the brain e. Yes, as the therapist you should not have begun treatment due to elevated BP pre-treatment

a. Yes, HTN is bad in the acute stage following stroke- you do want HTN in the acute stage b. Yes, this blood pressure is an abnormal response to light-moderate activity- you are not worried about this BP because you want blood flow to the brain c. No, this is not hypertensive- this is hypertensive d. No, you want HTN in the acute stage to allow blood flow to the brain- CORRECT- you want blood flow to the brain and do not want to use antihypertensives in the acute stage e. Yes, as the therapist you should not have begun treatment due to elevated BP pre-treatment - you are not worried because you want an elevated BP in the acute stage following stroke Generally, it's the first 24 hours of a stroke that you don't treat SBP<220. After that you gradually bring them down. I'll change the question accordingly.

You are at Walter Reed for your acute care rotation on the TBI floor. You are preparing for your next patient and perform a quick chart review. You see that your patient has been labeled as having a "severe TBI". In his chart you see the patient's abnormal CT and MRI results and recognize this as one criteria to grade a patient's TBI as "severe". What other factors result in a grading of "severe" in a patient following TBI? a. altered LOC < 30 mins, GCS 13-15, PTA < 24 hours b. LOC > 6 hours, GCS < 9, PTA > 7 days c. LOC < 6 hours, GCS 9-12, PTA < 7 days d. no LOC, GCS < 9, PTA 12 hours e. LOC > 6 hours, GCS 16-18, PTA 2-5 days

a. altered LOC < 30 mins, GCS 13-15, PTA < 24 hours - this is the classification for a Mild TBI b. LOC > 6 hours, GCS < 9, PTA > 7 days - CORRECT - this is the classification for a severe TBI c. LOC < 6 hours, GCS 9-12, PTA < 7 days - this is the classification for a moderate TBI d. no LOC, GCS < 9, PTA 12 hours - Although a GCS of < 9 does classify as a severe TBI, this LOC and PTA would classify as mild TBI. e. LOC > 6 hours, GCS 16-18, PTA 2-5 days - LOC is correct for a severe TBI, however, a GCS score of 16-18 does not exist, and PTA of 2-5 days would fall in the moderate TBI category

You patient, Jed I. Knight, is a 21 y/o male who got into a motorcycle accident after attempting to perform a wheelie. Among his numerous injuries, he suffered a TBI with initial loss of consciousness. Upon admittance to the hospital, images were taken showing skull fracture and epidural hematoma. Which of the following is NOT an extracranial factor that you would be concerned may contribute to SECONDARY brain injury? a. anemia b. malnutrition c. hyponatremia d. contusion in temporal lobe e. hypercapnia

a. anemia- this is a secondary injury of which you would want to be wary b. malnutrition- this is a secondary injury of which you would want to be wary c. hyponatremia- this is a secondary injury of which you would want to be wary d. contusion in temporal lobe- CORRECT this is a PRIMARY injury e. hypercapnia- this is a secondary injury of which you would want to be wary

Mrs. Ruth is a 52 y/o female you are evaluating in the NOA clinic. After greeting Mrs. Ruth in the waiting room, you observe her as she walks to the treatment plinth for the evaluation. Prior to asking any questions, you suspect that Mrs. Ruth may have a peroneal polyneuropathy because she demonstrates an obvious _________. a. antalgic gait b. steppage gait c. scissors gait d. wide base gait e. festinating gait

a. antalgic gait- indicates pain and is not suggestive of peroneal polyneuropathy b. steppage gait- CORRECT-steppage gait is typically seen in peroneal polyneuropathy c. scissors gait- is associated with spasticity as seen in Stroke, SCI and TBI patients d. wide base gait- is indicative of cerebellar ataxia e. festinating gait- is associated with Parkinson's Disease

Mr. Johnson is a 35 year old Caucasian male involved in a MVC 4 days ago, which resulted in a mild TBI. Before evaluating him in the acute care setting, you look back through your notes on TBI. You recall the most common areas affected by contusions and small hemorrhages from a TBI include? a. anterior and inferior frontal and occipital lobes b. anterior and inferior frontal and temporal lobes c. posterior and inferior occipital and temporal lobes d. anterior and inferior occipital and frontal lobes e. posterior and inferior frontal and occipital lobes

a. anterior and inferior frontal and occipital lobes: INCORRECT- the anterior inferior temporal lobe is part of the most common sites of b. anterior and inferior frontal and temporal lobes: CORRECT- the anterior inferior temporal lobe and anterior inferior frontal lobes are the most common areas affected by TBI becauses of their proximity to bony protrusions. c. posterior and inferior occipital and temporal lobes: INCORRECT- The posterior-inferior portions of the temporal and occipital are not the most common portion affected because the posterior portions are more likely to come in contact either with smoother bone, or not directly come in contact with bone at all. d. anterior and inferior occipital and frontal lobes: INCORRECT- the anterior inferior temporal is one of the most common areas affected from TBI, however the occipital lobe is not one of the most common areas because it is more likely to come in contact with smoother bone, therefore it is less likely to cause significant damage. e. posterior and inferior frontal and occipital lobes: INCORRECT- the anterior inferior frontal lobe is one of the most common areas affected from TBI, however the occipital lobe is not one of the most common areas because it is more likely to come in contact with smoother bone, therefore it is less likely to cause significant damage.

While on your acute care affiliation, you begin seeing an 18 y/o patient who sustained a TBI while incorrectly performing an intricate maneuver while intoxicated. He is now your patient and you read that he was graded as having severe disability on the Glasgow Outcome Scale. As you begin treating him, you become concerned about his slow progress, as you know he is at increased risk for developing all of the following except a. infections such as pneumonia or UTI b. venous thromboembolism c. complications due to previous medical problems d. secondary spinal cord injury e. delayed improvements and increased chance of secondary injuries

a. infections such as pneumonia or UTI- this is a concern you should be wary of when treating a TBI b. venous thromboembolism-- this is a concern you should be wary of when treating a TBI c. complications due to previous medical problems- this is a concern you should be wary of when treating a TBI d. secondary spinal cord injury- CORRECT- this is NOT a complication you would expect to develop during treatment of a TBI e. delayed improvements and increased chance of secondary injuries- intoxication at time of injury is associated with poorer outcome and increased chance of secondary injuries

It is the first day of your rehab affiliation at Magee on the TBI floor. Your CI has asked you to do an extensive chart review, and wants you to give her a summary of the chart by the end of the day. She also hands you the patient's MRI results and tells you that she has not had a chance to look them over but would like a summary of those as well. As you begin to look through the MRIs you notice a lot of different images. At first you panic because they all look that same, but luckily you remember your neuroimaging from UDPT and know that on T2 images CSF is ______, Flair images show ________, and DWI show ________. With these basics you are able to begin to differentiate the images and give your CI an educated summary by the end of the day. a. black; perfusion; ischemia b. white; edema; ischemia c. black; connections in the brain; active blood flow d. white; ischemia; edema e. white; edema; perfusion

a. black; perfusion; ischemia - Although DWI images show ischemia, on T2 images CSF is white (no black), and perfusion is shown on PWI images. b. white; edema; ischemia - CORRECT - on a T2 image water appears white, which would cause the image to show CSF as white. Flair images show edema, and DWI images aka diffusion weighted images show ischemia. c. black; connections in the brain; active blood flow - T2 images show CSF as white, connections in the brain are seen on DTI, and active blood flow is seen on fMRI. d. white; ischemia; edema - On T2 images CSF is white, but the rest of this answer choice is opposite from the correct answer. Flair = edema, DWI = ischemia e. white; edema; perfusion - Although T2 images so show CSF as white, and Flair images show edema, DWI images show ischemia NOT perfusion. Perfusion is seen on PWI.

Jordan is a 45 year old Chinese male, admitted to the hospital following a motorcycle accident. Despite wearing his helmet, Jordan had a 25 minute LOC and suffered a TBI. As an acute care PT specializing in trauma, you are wondering what Jordan's likely prognosis might be. All of the following are poor prognostic factors following TBI EXCEPT: a. decreased socioeconomic status b. low GCS score c. being 45 years old d. brainstem abnormalities e. prolonged PTA >8 weeks

a. decreased socioeconomic status: INCORRECT- Patients with decreased socioeconomic status tend to have a poorer outcomes following most pathologies. The reason for this is debated, but one of the theories is because they have decreased resources related to their recovery. b. low GCS score: INCORRECT- low score on the GCS, means the patient has lower level of consciousness and therefore requires a larger improvement to return to normal levels of consciousness. Having a GCS score </= to 5 24 hours after injury onset is predictive of a poor outcome. c. being 45 years old: CORRECT- the cutoff age with typically poorer outcomes begins at ages >55 years old d. brainstem abnormalities: INCORRECT- the brainstem is a primal part of the brain important in primal behaviors such as cardiac function and breathing. Abnormalities of any of these primal behaviors lead to a decreased prognosis because they are vital for life. e. prolonged PTA >8 weeks: INCORRECT- >12 weeks of PTA is a strong prognostic predictor for poor recovery, whereas <2 weeks indicates a good predictor of good recovery

Mr. Voltage is a 46 year old male post T6 SCI you are treating in the spinal cord unit of your acute care hospital. During a treatment session one week post injury, you suspect the patient to be displaying signs of autonomic dysreflexia. Which of the following signs and symptoms is NOT a sign of autonomic dysreflexia? a. headache b. tachycardia c. bradycardia d. diaphoresis e. hypertension

a. headache -- symptom of AD. b. tachycardia -- CORRECT. NOT a symptom of AD due to parasympathetic involvement in the correction of the sympathetic response associated with AD. c. bradycardia-- the sympathetic response of hypertension stimulates baroreceptors in the carotid sinuses and aortic arch. The parasympathetic nervous system attempts to maintain homeostasis by slowing down the heart rate but is unable to counteract these effects through the injured spinal cord. The brainstem stimulates the heart through the vagus nerve, causing bradycardia. d. diaphoresis -- result of the sympathetic response to AD. e. hypertension -- result of the sympathetic response to AD.

Mr. Knight (from question 8), upon injury, was rushed to your hospital for acute management. The doctors decide to monitor his ICP, but cannot agree on what other treatments would be best. Because you are an amazing PT, you suggest adding the following treatment: a. hyperbaric oxygen, which has been shown to improve outcomes both acutely and chronically b. routine use of steroids, as this will help delay atrophy c. hypothermia, because studies suggest neuroprotective effects d. neurodestructive medications, as TBIs cause brain damage that is corrected with these medications e. nothing, TBIs resolve on their own

a. hyperbaric oxygen, which has been shown to improve outcomes both acutely and chronically- hyperbaric oxygen does NOT improve outcomes, either acutely or chronically b. routine use of steroids, as this will help delay atrophy- routine use of steroids is not recommended, as this is associated with increased risk of death and disability c. hypothermia, because studies suggest neuroprotective effects- CORRECT- this is a true statement d. neurodestructive medications, as TBIs cause brain damage that is corrected with these medications- hypothermia combined with neuroPROTECTIVE medications may be beneficial e. nothing, TBIs resolve on their own- the best form of treatment for acute TBI is not to do nothing

You recently were scheduled a new 22 year old male patient who sustained a TBI from a MVA. You receive the imaging completed in the trauma department and decide to take a closer look. You notice his contusions and specifically locate the coup and contrecoup. What is the meaning of these two locations respectively? a. injuries in the epidural space; injuries in the intraparenchymal space b. injuries away from the site of impact; injuries at the site of impact c. injuries in the subdural space; injuries in the epidural space d. injuries at the site of impact; injuries away from the site of impact e. injuries in the intraparenchymal space; injuries in the subdural space

a. injuries in the epidural space; injuries in the intraparenchymal space-INCORRECT- while contusions at the coup and contrecoup could be in these locations there are many other locations indicated as well and this is not the meaning of these two terms b. injuries away from the site of impact; injuries at the site of impact- INCORRECT- coup and contrecoup location definitions have been switched here c. injuries in the subdural space; injuries in the epidural space-INCORRECT- while contusions at the coup and contrecoup could be in these locations there are many other locations indicated as well and this is not the meaning of these two terms d. injuries at the site of impact; injuries away from the site of impact-CORRECT- coup and contrecoup locations respectively mean injuries at the site of impact; injuries away from the site of impact e. injuries in the intraparenchymal space; injuries in the subdural space-INCORRECT- while contusions at the coup and contrecoup could be in these locations there are many other locations indicated as well and this is not the meaning of these two terms

Mrs. Norris is a 72 y/o female with diabetes you are treating on your acute care affiliation. During her initial evaluation, you note Mrs. Norris has diminished sensation in her extremities. When asked if she experiences any numbness or tingling, she describes numbness and tingling in the stocking and glove distribution. Which peripheral nerve pattern does this represent? a. mononeuropathy b. mononeuropathy multiplex c. polyneuropathy d. plexopathy e. radiculopathy

a. mononeuropathy - more than one peripheral nerve is affected in this patient b. mononeuropathy multiplex - stocking and glove is not a random distribution c. polyneuropathy- CORRECT- this is characterized by symmetrical, distal greater than proximal and is often present in diabetic neuropathies d. plexopathy - the retroperitoneal or brachial region nerve complex would be affected and will not present in this sensory loss distribution e. radiculopathy - stocking and glove does not follow a particular nerve root distribution

Mrs. Brown is a 76 year-old female presenting to outpatient PT s/p a major ischemic stroke affecting the R MCA distribution and therefore, the L UE. Her family is extremely concerned that she will not be able to use her L UE again and would like to know if functional recovery should be expected. As an educated PT you are aware of two indicators for a poor prognosis that you can easily observe/test: a. Reflex testing/Sensation testing of the L UE b. MMT of the UE/ Passive ROM of UE c. MMT of the UE/ grip strength d. Sensation testing of L UE/ Muscle tone e. MMT of the LE/mini mental exam

a. reflex testing/sensation testing of the L UE- these would test nerve root pathology and not likely be affected with a stroke b. MMT of the UE/Passive ROM of UE- MMT of the UE is correct because it looks for voluntary movement but passive ROM would test for joint pathology/injury c. MMT of the UE/grip strength-CORRECT -all of these tests look for voluntary movement and likely to be affected by a stroke to the MCA d. Sensation testing of the L UE/muscle tone- sensation testing would look at nerve root pathology and muscle tone changes may indicate improvement if muscle tone increases but MMT for movement is the most correct answer e. MMT of the LE/mini mental exam- MMT of the LE would not show improvement with the UE nor would the mini mental exam

While on your acute care affiliation you are assigned a 35 year old female TBI patient. Upon entering the room the nurse tells you this patient has recently gone from vegetative to vigilant. You immediately think back to your medical science class and remember this means.... a. restraints and antipsychotics will be a necessary intervention because the patients poses an immediate threat to self or others b. the patient's spontaneous eye opening and sleep-wake cycles have been restored c. scheduled multimodality stimulation and monitoring to assess responsiveness would not be an appropriate intervention d. the patient has lost the ability to regulate their sleep-wake cycles e. neurostimulant medication and minimizing other Rx side effects would not be an appropriate intervention

a. restraints and antipsychotics will be a necessary intervention because the patients poses an immediate threat to self or others-INCORRECT- restraints and antispychotics would only be indicated if the patient was exhibiting behavioral issues and posttraumatic delirium b. the patient's spontaneous eye opening and sleep-wake cycles have been restored -CORRECT- vegetative to vigilant means the patient has regained the ability to regulate their sleep-wake cycles c. scheduled multimodality stimulation and monitoring to assess responsiveness would not be an appropriate intervention- INCORRECT- this intervention would be appropriate for a patient who has gone from vegetative to vigilante d. the patient has lost the ability to regulate their sleep-wake cycles=INCORRECT- vegetative to vigilant means the patient has regained the ability to regulate their sleep-wake cycles e. neurostimulant medication and minimizing other Rx side effects would not be an appropriate intervention- INCORRECT- this intervention would be appropriate for a patient who has gone from vegetative to vigilante

Sir Amyloid is a 54 year old male from Switzerland. He has come all this way to see you as he has heard you are an expert in neurology. During your examination you find that he is able to maintain a Romberg stance for 30 seconds with EO, but is unable to perform this test with EC. You have been informed that he has had a spinal cord injury, but he has no imaging to assist you during your evaluation. However, based on your examination findings, you believe he likely has an injury to which spinal tract? a. Spinocerebellar tract b. Spinothalamic tract c. Reticulospinal tract d. Tectospinal tracts e. Rubrospinal tract

a. spinocerebellar -- CORRECT -- This tract relays proprioceptive feedback from neuromuscular spindles and GTOs of ipsilateral trunk and LE to the cerebellum and plays an important role in balance. b. Spinothalamic tract -- incorrect -- transmits non discriminate touch, pain and temperature sensations from the body and is not as important with EC balance c. Reticulospinal tract -this tract is involved in spinal reflexes and pain transmission d. Tectospinal tracts- this tract plays a role in head turning to sudden auditory or visual stimuli e. Rubrospinal tract- this tract is involved in motor function of the upper extremity.

Mrs. Smith is a 50 year old female post SCI with cervical neck pain who you have been treating at your outpatient clinic for three weeks. So far, little progress has been made towards reaching her LTGs and this last treatment her symptoms of back pain and numbness and tingling have gotten worse. You make the best clinical decision and refer her to a physician for follow-up. Which of the following would NOT be an indication for this patient to have surgery following her SCI? a. spondylosis b Chiari malformation c. malignant tumor d. Osteophyte and ligament malformations e. Impaired sensation to L3 dermatome

a. spondylosis -- may have surgery to prevent further progression of malformation b Chiari malformation - surgery to prevent progression of non-communicating hydrocephalus. c. malignant tumor - surgery to prevent progression of cancer and further damage to the cord. d. Osteophyte and ligament malformations - remove harmful stimuli to the cord. e. Impaired sensation to L3 dermatome - CORRECT this is not an indication for surgery and patient will benefit from PT for some possible recovery of sensation.

Susan Foyle is 75 y.o patient that you are treating today for the first time. She was hospitalized after falling in her home (due to an unsecured throw rug) and experiencing intracranial bleeding among other orthopedic injuries. Given the mechanism of injury and age of the patient, before you see the CT you suspect a _______________ bleeding pattern. a. subdural b. epidural c. biconvex d. intraparenchymal e. enlarged ventricles

a. subdural- CORRECT- this is the bleeding pattern associated with elderly falls b. epidural- this is a bleeding pattern, but is not associated with elderly falls c. biconvex- this is the same as a epidural bleeding pattern d. Intraparenchymal- this bleeding pattern can be seen in the aged adult, but given the MOI it is unlikely in this scenario e. Enlarged ventricles- this is a sign of aging but would not be of major concern

You and your CI are working with a 18 year old male patient who sustained a TBI after he was hit by a car. Your CI wants to make sure you are well informed on the physiology of his injury so she begins to probe you for details of this DIA. She asks you to identify the patient's contusion on his imaging and asks you to explain the the mechanism of this injury, so you tell her... a. this injury occurs because of shear forces occurring at the interface of gray matter and the skull due to the differences in densities b. this injury occurs because the temporal horns dilate and induce hydrocephalus c. this injury occurs because of shear forces occurring at the interface of white and gray matter due to the differences in densities d. this injury occurs because of severe hypertension resulting in an intraparenchymal bleed e. this injury occurs because of shear forces occurring at the interface of white matter and the skull due to the differences in densities

a. this injury occurs because of shear forces occurring at the interface of gray matter and the skull due to the differences in densities-INCORRECT- this injury occurs at the interface of gray and white matter b. this injury occurs because the temporal horns dilate and induce hydrocephalus-INCORRECT- while the temporal horns tend to dilate first with hydrocephalus, this is not the mechanism of injury for a contusion in the brain c. this injury occurs because of shear forces occurring at the interface of white and gray matter due to the differences in densities-CORRECT- this is the mechanism of injury for a contusion in the brain d. this injury occurs because of severe hypertension resulting in an intraparenchymal bleed-INCORRECT- while the most common cause of intraparenchymal bleeds is hypertension, this is not the mechanism of injury for a contusion in the brain e. this injury occurs because of shear forces occurring at the interface of white matter and the skull due to the differences in densities-INCORRECT- this injury occurs at the interface of gray and white matter.

Mrs. Ross is a 62 y/o Caucasian female you are treating in the NOA clinic for a peroneal polyneuropathy secondary to alcoholism. During ambulation, Mrs. Ross demonstrates a steppage gait as though she is walking up stairs, despite the fact she is on a flat surface. Upon further examination of the patient, you find she is unable to a. walk on her heels b. walk on with heels off ground c. feel her medial calf d. swallow e. extend her knee

a. walk on her heels - CORRECT- Anterior compartment muscles producing dorsiflexion and innervated by an injury to the common peroneal nerve associated with the steppage gait pattern would be affected by the injury b. walk with heels off ground - Posterior compartment muscles producing plantar flexion are innervated by the tibial nerve and would not be affected by an injury to the common peroneal nerve (may not be able to walk on toes) c. feel her medial calf - sensation to the medial calf is supplied by the saphenous nerve and would not be affected by an injury to the common peroneal nerve. d. swallow - CN IX and X innervate the smooth esophageal muscle responsible for swallowing and would not be affected by a peripheral nerve injury to the common peroneal nerve. e. Extend her knee - extension of the knee is an action produced by the quadriceps muscles innervated by the femoral nerve.


Kaugnay na mga set ng pag-aaral

Reading 48- Overview of Equity Securities

View Set

Lord of the Flies Chapter 10 - The Shell and the Glasses

View Set

Chapter 39: Oxygenation and Perfusion

View Set

Intro to Anthropology, Psychology and Sociology Definitions

View Set

Interconnecting Cisco Network Devices Part 1 (ICND1) v2.0 Set 2

View Set